Archive for the ‘Biblical errancy’ Category

h1

Failed prophecies?

July 3, 2014

Prophecies are consistently appealed to in the Bible as evidence of its divine origin. Several passages claimed by skeptics and atheists as examples of failed prophecies are examined here. They fall into the following categories: passages which are not formal predictive prophecies in the manner claimed, passages which are misinterpreted by the critic, and passages which were fulfilled in contradiction to the critic’s claims.

Passages which are applied typologically

Several passages in the Old Testament which have  traditionally been understood as prophetic of Christ, are objected to by critics as inapplicable or unfulfilled.

  1. How can ‘When he sins, I will correct him with the rod of men and with wounds inflicted by human beings’ in 2 Samuel 7:14 apply to Christ, when Christ never sinned?
  1. How can the servant of Isaiah 49 refer to Christ, when the context indicates it speaks explicitly of Israel?

A traditional answer has been that these prophecies have ‘dual fulfillments’, that they were prophecies fulfilled in part by Solomon or Israel, and fulfilled in part later in the life of Christ. This is actually only half true. In reality the prophecies had direct and full application to their immediate referent, and are applied typologically to Christ. That is, they are formal predictive prophecies about Solomon and Israel respectively, but parts of them find echoes in the life of Christ, the ‘son of God’ in a greater sense than both Solomon and Israel. Their application to Christ is not a claim to direct fulfillment of a formal prediction, but an illustration that certain promises made by God in earlier times have a relevance to the greater work of Christ which they foreshadow.

Passages which are misinterpreted

Certain prophecies criticized as unfulfilled have actually been misinterpreted.

  1. How can God’s words  ‘I, the LORD, promise: “David will never lack a successor to occupy the throne over the nation of Israel’ in Jeremiah 33:17 be said to have been fulfilled, when the Davidic monarchy ended with the Babylonian captivity?

The answer is that this prophecy was conditional, as was made clear to Solomon before Jeremiah’s time.

1 Kings 9:

You must serve me with integrity and sincerity, just as your father David did. Do everything I commanded and obey my rules and regulations.

Then I will allow your dynasty to rule over Israel permanently, just as I promised your father David, ‘You will not fail to have a successor on the throne of Israel.’

Solomon and his descendants failed to keep this covenant, and the Davidic monarchy consequently came to an end;[1] the New Testament writers identify Christ as its true successor.

  1. How can Jesus’ words ‘I tell you the truth, you will not finish going through all the towns of Israel before the Son of Man comes’ in Matthew 10:23 and ‘this generation will not pass away until all these things take place’ in Matthew 24:34 be considered anything but failed prophecies, when he did not return before the apostles had finished preaching to the towns of Israel, or before the generation of his time had passed away?

These passages do not speak of the return of Christ, but of the end which would come with the destruction of Jerusalem in the war of 66-70 CE.[2] [3]

  1. In Mark 8:38 Jesus says ‘there are some standing here who will not experience death before they see the kingdom of God come with power’, and in Matthew 16:28 Jesus says ‘there are some standing here who will not experience death before they see the Son of Man coming in his kingdom’, Jesus failed to return before his disciples died, making these false prophecies.

These passages refer to thetransfiguration (a vision of Jesus ‘coming in his kingdom’), which is described in both gospels as occurring shortly after Jesus spoke these words. Early Christians almost universally understood these passages as a reference to the transfiguration.[4]

It should be noted that these words of Jesus were transmitted by the gospel writers decades after Jesus’ ministry and after the disciples had died, indicating that they them as fulfilled prophecy. If these predictions had been understood as Jesus saying he would return before the disciples died, they would have been more likely to omit them completely.

  1. In 1 Corinthians 7:31 Paul says ‘the present shape of this world is passing away’, indicating that he, like other New Testament writers, believed they would see the return of Christ in their lifetime, but it never happened.

These words are not written as formal predictive prophecy, and as much as Paul may have believed at one time that Christ would return while he was still alive,  it is clear he and other writers such as Peter gradually understood that this would not be the case. In fact both Paul and Peter prepare other Christians for a lengthy wait before Jesus’ return. In 2 Thessalonians Paul makes it clear that Christ’s return is not imminent, and that it will not happen before certain specific events have taken place.[5]

2 Thessalonians 2:

1 Now regarding the arrival of our Lord Jesus Christ and our being gathered to be with him, we ask you, brothers and sisters,

2 not to be easily shaken from your composure or disturbed by any kind of spirit or message or letter allegedly from us, to the effect that the day of the Lord is already here.

3 Let no one deceive you in any way. For that day will not arrive until the rebellion comes and the man of lawlessness is revealed, the son of destruction.

Peter likewise prepares his readers for a lengthy wait, speaking of the ‘last days’ in the future, and warning believers they will be mocked for their patience at that time.

2 Peter 3:

3 Above all, understand this: In the last days blatant scoffers will come, being propelled by their own evil urges

4 and saying, “Where is his promised return? For ever since our ancestors died, all things have continued as they were from the beginning of creation.”

Passages which were fulfilled

Certain prophecies have actually been fulfilled, despite claims to the contrary.

  1. The prophecy of Isaiah 17 claims Damascus would be ‘a heap of ruins (verse 1), and that ‘Damascus will lose its kingdom’ (verse 23), but Damascus is a thriving city today.

This prophecy made by Isaiah some time around 740 BCE, was fulfilled when the Assyrian king Tiglath-pileser III destroyed Damascus, which was at that time not a mere city but a thriving Aramean kingdom.[6] Isaiah prophesied Damascus would be a ‘heap of ruins’, and Tiglath-pileser III’s own record of his conquest proves this is what happened, boasting of his extensive destruction of Damascus; ‘591 cities of the 16 districts of Damascus I destroyed like mounds of ruins after the Deluge’. [7] Isaiah’s prediction that ‘Damascus will lose its kingdom’ was fulfilled by Tiglath-pileser III’s annexation of Damascus and all its territories, and the deportation of  many of its inhabitants.[8] [9]

  1. The prophecy of Tyre’s destruction in Ezekiel 26 failed to come true, and Ezekiel’s own words in Ezekiel 29:18-19 (especially that Nebuchadnezzar II and his army ‘received no wages from Tyre for the work he carried out against it’ , verse 28), prove he knew the prophecy failed.

This criticism overlooks the fact that Ezekiel’s commentary in Ezekiel 29:18-19 is certainly in response to criticisms that his earlier prophecy was not completely successful; in other words, it proves at minimum that Ezekiel’s earlier prediction that Nebuchadnezzar II would attack Tyre was not only made before the event, but also came true. This is not a good start for a critic of prophetic fulfillment. Subsequent to Ezekiel’s successful prediction, his political enemies attempted to discredit him by claiming Nebuchadnezzar’s army had failed to  vindicate the prophecy.[10] Such claims were unwarranted, for ‘the siege was successful and Tyre did pass into Babylonian control’.[11]

Since Nebuchadnezzar II failed to destroy Tyre utterly, some commentators state that since Ezekiel’s original prophecy had predicted ‘many nations’ would be brought against Tyre (‘I will bring up many nations against you’, Ezekiel 26:3), of which Nebuchadnezzar II’s campaign was only the first. It is claimed that this is supported by the alternating statements of what ‘he’ (Nebuchadnezzar II), and ‘they’ (subsequent nations), would do to Tyre, especially Alexander the Great.

However, this view has been criticized as an improbable reading of the Hebrew text.[12] Instead it should be recognized that the first section of the prophecy (verses 1-6), is a self-contained unit predicting the coming of ‘many nations’ against Tyre, and thus not restricted to Nebuchadnezzar II (whose campaign is described from verses 7-13);[13] the complete destruction of the city as predicted by Ezekiel was fulfilled by Alexander.

The objection that the prophecy’s description of an attack against a mainland city (which Nebuchadnezzar attacked), and therefore does not describe the destruction of the island (which Alexander attacked), is misguided. [14] The prophecy uses the standard conquest language of the Ancient Near East; when Esarhaddon of Assyria attacked the island city he still described it in terms of a land battle, even to the point of describing trenches being dug (impossible in an island siege).[15]

  1. The prophecies against Egypt in Ezekiel 30 and Isaiah 19 failed to come true; Nebuchadnezzar did not invade and conquer Egypt as predicted.

First it should be noted that it is acknowledged even by modern critics that the prophecy was given before the event it describes took place.[16] Consequently, attempts to reduce the accuracy of the prediction by claiming it was insufficiently fulfilled are demonstrably motivated by the desire to avoid the fact that a successful prophecy actually happened. The best a critic can do in the face of the fact that the text contains a prophecy indisputably before the event to which it obviously refers, is to claim that the prophecy wasn’t fulfilled sufficiently to be considered accurate. This is not intellectually honest.

In fact, the prophecy was fulfilled by Nebuchadnezzar II’s war against Egypt in 586 BCE, recorded in a fragmentary Babylonian text.

‘. . . [in] the 37th year, Nebuchadnezzar, king of Bab[ylon] mar[ched against] Egypt (Mi-sir) to deliver a battle. [Ama]sis (text: [ . . . ]-a(?)-su)y of Egypt, [called up his a]rm[y] . . . [ . . . ]\u from the town Pufu-Iaman . . . distant regions which (are situated on islands) amidst the sea . . . many . . . which/who (are) in Egypt . . . [car]rying weapons, horses and [chariot]s . . . he called up to assist him and . . . did [ . . . ] in front of him . . . he put his trust. . . (only the first signs at the beginning and the end of the following 7 or 8 lines are legible).’[17]

The success of Nebuchadnezzar’s campaign is acknowledged by secular historians, on the basis of several lines of evidence.[18] Firstly, and rather ironically, some historians believe the prophecy of Ezekiel was written after the event, precisely because it is so accurate.

‘First of all, both Ezekiel and Jeremiah prophesied that he would do so; and since most of these “prophecies” were written in retrospect, or at least gained popular currency only after having been proved correct, we may be fairly certain that the prophesied invasion and defeat of Egypt actually took place.’[19]

Two other sources are the Biblical text describing Jewish refugees moved from Egypt to Babylon, and the record of Josephus indicating Nebuchadnezzar defeated Egypt.

‘Secondly, the biblical sources say that Nebuchadrezzar was able to remove the Jewish refugees in Egypt to Babylon. He could not of course have done so unless he had entered and subjugated the country. Thirdly, Josephus tells us that he conquered Egypt. We are informed that four years after the fall of Tyre, Nebuchadrezzar invaded the country and put its King Uaphris to death, installing a creature of his own upon the vacant throne.’[20]

Another source is the presence in Egypt of artifacts belonging to Nebuchadnezzar II, demonstrating he invaded and established himself there.

‘Fourthly, and most importantly, artifacts of Nebuchadrezzar have actually been discovered in Egypt. These are “three cylinders of terra-cotta bearing an inscription of Nebuchadnezzar, an ordinary text referring to his constructions in Babylon … These were said to come from the Isthmus of Suez, and they apparently belong to some place where Nebuchadrezzar had ‘set up his throne’ and ‘spread his royal pavilion.’ As he only passed along the Syrian road, and Daphnae would be the only stopping place on that road in the region of the isthmus, all the inferences point to these having come from Defenneh, and being the memorials of establishment there.”’ [21]

This also fulfills the prophecy of Jeremiah 43:10 that Nebuchadnezzar would ‘pitch his royal tent’ in Tahpanhes in Egypt.[22] [23] The scholarly  conclusion from these sources is ‘There can be little doubt; Nebuchadrezzar entered and conquered Egypt.’ [24]

  1. Although Joshua 3:10 says God would ‘truly drive out before you the Canaanites, Hittites, Hivites, Perizzites, Girgashites, Amorites, and Jebusite’, 1 Kings 9:20 says ‘several non-Israelite peoples were left in the land after the conquest of Joshua’.

Joshua’s words are a repetition of the words of Moses, which made it clear that God’s driving out of the  inhabitants of Canaan was conditional on the Hebrews maintaining their obedience to God.

Deuteronomy 4:

1 Now, Israel, pay attention to the statutes and ordinances I am about to teach you, so that you might live and go on to enter and take possession of the land that the LORD, the God of your ancestors, is giving you.

Deuteronomy 7:

12 If you obey these ordinances and are careful to do them, the LORD your God will faithfully keep covenant with you as he promised your ancestors.

Deuteronomy 8:

1 You must keep carefully all these commandments I am giving you today so that you may live, increase in number, and go in and occupy the land that the LORD promised to your ancestors.

  1. The gospels describe Jesus predicting the destruction of Jerusalem and the Temple, but this is not an accurate prediction because the gospels were written after the event; even if they had been written before the event, such a war was obviously going to happen anyway, so it is not evidence of an accurate prophecy.

The first point to note about this objection (which can be found made by a range of atheists and skeptics), is that it is intellectually dishonest; it tries to argue that the prophecy isn’t true because it was written after the event, but also argues that even if it had been made before the event it still wouldn’t count as a fulfilled prophecy. Such an argument is not evidence based, and demonstrates that the person making the argument is not really interested in the facts.

The second point to note about this objection is that although the date of Mark’s gospel is still an open question in current scholarship, dates proposed typically fall between 65 and 75 CE.[25] In fact recently strong arguments have been made for a much earlier date. The secular scholar Mark Crossley argues for a date ‘before the late forties’,[26] at least 20 years before the destruction of Jerusalem, and the secular scholar Maurice Crossley argues ‘a date c. 40 CE must be regarded as highly probable’.[27]

It is clear that many scholars (even non-religious scholars), are prepared to accept that Mark’s gospel (which contains a prophecy of the destruction of Jerusalem and the Temple), was indeed written before the events it predicts. Rejecting this possibility out of hand is therefore intellectually dishonest, especially when no attempt is made to address the significant body of evidence indicating that Jesus’ prediction in Mark was indeed made before the event.

The third point to note is that there is no evidence at all to suggest that the possibility of the destruction of Jerusalem and the Temple by the Romans was so probable that it would have appeared obvious to people in Jesus’ own day, or even 30 years later (63-65 CE). In fact construction on the Temple had already been carried out for nearly 50 years by the time of Jesus’ ministry (John 2:20), and it was not even completed until shortly before the Romans destroyed it.[28][29]

It is difficult to demonstrate that anyone seeing the ongoing construction of the Temple in Jesus’ day would have concluded that the Romans would destroy it over 30 years later. It is likewise difficult to demonstrate that anyone seeing the construction continuing in 63 CE would have concluded that the Romans would destroy both the Temple and the entire city in just a few more years.

The fact is that the explosive events which led to the destruction of Jerusalem in 70 CE were not predicted by anyone outside the gospels, and even Jesus’ own words make it clear he expects skepticism on the part of his audience. There is no evidence in any of the relevant Roman historical sources who wrote in detail about the events of the first century (such as Suetonius, Tacitus, and Appian), nor in Josephus (who was both a historian, and a general on the Jewish side of the war), that anyone was expecting such a war before it took place, still less the complete destruction of Jerusalem and the Temple. There is excellent evidence therefore that Jesus’ prophecy was made well before the events they predicted successfully, and it is indisputable that these events were not foreseen or expected by anyone else.

Conclusion

Skeptical claims that Bible prophecies have not been fulfilled should be taken seriously; it is a fact that some Bible prophecies are impossible to verify due to a lack of available information, even if there is no evidence proving they did not come to pass. However, when such criticisms are made it is important to identify whether or not the arguments made are evidence based, demonstrate a knowledge of and engagement with the relevant scholarly literature, and are intellectually honest.

In particular, such arguments must prove that they have interpreted the prophecy as it was originally intended, and must provide substantial objections to the relevant counter-arguments by scholars who make the case that the prophecy was in fact fulfilled. In turn, defenses of these prophecies must exercise intellectual honesty in acknowledging problems where they genuinely exist, and must cite, discuss, and be supported by the relevant scholarly literature, in order to be credible.

____________________________________

[1] ‘But perhaps the promise to the house of David is not all that unilaterally unconditional, and perhaps the dynastic oracle here is firmly set in the Sinaitic covenant (Eslinger 1994). We can agree with Brueggemann (1990: 259) when he says, “While the covenantal ‘if’ is silenced in this theology, it has not been nullified.” This is particularly true when the Lord speaks of David’s son: “When he commits iniquity, I will punish him with a rod such as mortals use, with blows inflicted by human beings” (v. 14b).’, Victor P. Hamilton, Handbook on the Historical Books (Grand Rapids, MI: Baker Academic, 2001), 317–318.

[2]Now that we have seen that the reference is to the destruction of the temple, which did as a matter of fact take place some 40 years later while many of Jesus’ contemporaries must have been still alive, all such contrived renderings may be laid to rest. This verse refers to the same time-scale as 16:28 (which was also concerned with the fulfillment of Dan 7:13–14): “some of those standing here will certainly not taste death before …” (cf. also 10:23, with the same Daniel reference: “you will not go through all the towns of Israel before …”).’, R. T. France, The Gospel of Matthew (The New International Commentary on the New Testament; Grand Rapids, MI: Wm. B. Eerdmans Publication Co., 2007), 930.

[3]34 The πάντα ταῦτα, “all these things,” of this verse can include no more than the same phrase in the preceding verse and thus cannot include the coming of the Son of Man (so too Blomberg). The phrase refers not only to general marks of the interim period such as tribulation, distress, pseudo-messiahs, and false prophets but specifically, and dramatically, to the desecration of the temple and the destruction of Jerusalem (cf. vv 15–22). As in the other imminence sayings (cf. 16:28; 10:23; 23:36), all of which like the present logion are prefaced by the emphatic ἀμὴν λέγω ὑμῖν, “truly I tell you,” formula, the main point is that the fall of Jerusalem was to be experienced by that generation (pace Kidder), those listening there and then to the teaching of Jesus (ἡ γενεὰ αὕτη, “this generation,” is used consistently in the Gospel to refer to Jesus’ contemporaries; cf. 11:16; 12:41–42, 45; 23:36).’ Donald A. Hagner, Matthew 14–28 (vol. 33B; Word Biblical Commentary; Dallas: Word, Incorporated, 1998), 715.

[4]‘The most widespread interpretation in the Eastern and Western church related the saying to the transfiguration whereby then “some” referred to Peter, James, and John.’, Ulrich Luz, Matthew: a Commentary (ed. Helmut Koester; Hermeneia—a Critical and Historical Commentary on the Bible; Minneapolis, MN: Augsburg, 2001), 386; Luz disagrees with this interpretation, but acknowledges it was the most common in the early church.

[5] ‘Mearns (“Development”) also takes the reference to be to 1 Thessalonians, but argues that Paul had changed his mind about the suddenness of the Day of the Lord since writing that letter and is now correcting perfectly reasonable inferences that the readers might have drawn from it.’, F. F. Bruce, 1 and 2 Thessalonians (vol. 45; Word Biblical Commentary; Dallas: Word, Incorporated, 1998), 164.

[6] ‘The Assyrian invaded northern Israel (2 K. 15:29) and then besieged and destroyed Damascus as an Aramean kingdom in 732, killing Rezin (2 K. 16:9). Assyrian records tell of 591 towns of the “16 districts of Aram” destroyed “like mounds left by a flood” (ARAB, I § 777).’, M. F. Unger, “Damascus,” ed. Geoffrey W. Bromiley, The International Standard Bible Encyclopedia, Revised (Wm. B. Eerdmans, 1979–1988), 854.

[7] ‘12′ […] orchards without number I cut down; I did not leave a single one. 13′ … the town of …]hadara, the home of the dynasty of Rezin of Damascus, 14′ [the pl]ace where he was born, I surrounded and captured. 800 people with their possessions 15′ their cattle (and) their sheep I took as spoil. 750 captives from the cities of Kuruṣṣa 16′ (and) Sama, 550 captives from Metuna I took, 591 cities 17′ of the 16 districts of Damascus I destroyed like mounds of ruins after the Deluge.’, Lester L. Grabbe, “The Kingdom of Israel from Omri to the Fall of Samaria: If We Only Had the Bible …,” in Ahab Agonistes: The Rise and Fall of the Omri Dynasty (ed. Lester L. Grabbe; London: T&T Clark, 2007), 79.

[8] ‘Finally, the Assyrians had had enough of the rebellious behaviour of Damascus, the last Aramaean stronghold in Hatti. Damascus and its cities were conquered and turned into Assyrian provinces (Tadmor 1994: 79–81). A part of the population was deported (Grayson 1991/2000: 77–8; Dion 1997: 215–16; Sader 1987: 250–1; Weippert 1987: 99).’, Hans M. Barstad, “Can Prophetic Texts Be Dated? Amos 1–2 as an Example,” in Ahab Agonistes: The Rise and Fall of the Omri Dynasty (ed. Lester L. Grabbe; London: T&T Clark, 2007), 33.

[9] ‘At the close of the Syro-Ephraimitic War, Tiglath-pileser took several actions that form the background of this text. He killed Rezin, destroyed Damascus, and annexed all territory controlled by Damascus into the Assyrian provincial system.‘, Brad E. Kelle, “What’s in a Name? Neo-Assyrian Designations for the Northern Kingdom and Their Implications for Israelite History and Biblical Interpretation,” ed. Gail R. O’Day, Journal of Biblical Literature 121 (2002): 639., Gail R. O’Day, ed., Journal of Biblical Literature 121 (2002): 659.

[10] ‘The objective reason for the oracle is supplied in v 18. However, the more immediate agenda is implied by v 21ab: Ezekiel was being criticized by his Jewish contemporaries for the lack of precise fulfillment of his oracles against Tyre.’. Leslie C. Allen, Ezekiel 20–48 (vol. 29; Word Biblical Commentary; Dallas: Word, Incorporated, 1998), 109.

[11] ‘It was to some extent a carping criticism: the siege was successful and Tyre did pass into Babylonian control. In a list of royal hostages at Nebuchadnezzar’s court, to be dated about 570 B.C., the king of Tyre has the initial place (ANET 308a; Katzenstein, History of Tyre 326). About 564 B.C. Baal, Ethbaal’s successor as king of Tyre, was replaced by a Babylonian High Commissioner (Katzenstein, History 332–33; cf. Unger, ZAW 44 [1926] 314–17). Any prophet might have been glad to chalk it up as a vindication of his or her prediction, despite Nebuchadnezzar’s non-destruction of Tyre.’, ibid., p. 109.

[12] ‘However, the proposed distinction between the “many nations” of verse 3 and Nebuchadnezzar’s army seems overly subtle, in light of the reference to Nebuchadnezzar as “king of kings” (v. 7) and the multiethnic nature of his army. Nebuchadnezzar is the focal point of verses 7–11, but the actions described are those of an army. The subject of the plural forms in verse 12 is most naturally understood as the collective “army” (Heb. ‘am) of verse 7, which in turn can be seen as comprised of the “many nations” mentioned in verse 3 (see also the reference to “nations” in v. 5).’, Robert B. Chisholm Jr., Handbook on the Prophets: Isaiah, Jeremiah, Lamentations, Ezekiel, Daniel, Minor Prophets (Grand Rapids, MI: Baker Academic, 2002), 268.

[13]Tyre’s fall appears in the first two prophecies, the second picking up phrases from the first and adding further details, and the remaining two prophecies describe the bewailing and then the entombment of the fallen one.’, Ronald M. Hals, Ezekiel (vol. 19; The Forms of the Old Testament Literature; Grand Rapids, MI: William B. Eerdmans Publishing Company, 1989), 188.

[14]Taking the standard siege imagery too literally, some scholars have concluded that the passage must come out of a setting different from Nebuchadrezzar’s time, seeing in vv. 9–11 a battle song about Alexander’s conquest of Tyre, or supposing that the actual reference of the passage was to a conquest of “old Tyre” on the adjacent mainland. (See the references in Zimmerli, Ezekiel 2, 37.) The proper recognition of the typicality of the imagery involved renders such literalistic hypotheses unnecessary.’, Ronald M. Hals, Ezekiel (vol. 19; The Forms of the Old Testament Literature; Grand Rapids, MI: William B. Eerdmans Publishing Company, 1989), 189.

[15] ‘The infiltration of characteristics typical of a description of a mainland siege into the description of the siege of the island city of Tyre can already be observed in the Assyrian royal inscriptions, when Esarhaddon there reports of the construction of trenches (ḫalṣē) against Tyre.’, Walther Zimmerli, Frank Moore Cross, and Klaus Baltzer, Ezekiel: a Commentary on the Book of the Prophet Ezekiel (Hermeneia—a Critical and Historical Commentary on the Bible; Philadelphia: Fortress Press, 1979–), 37; a footnote adds ‘This also disposes of Wiener’s theory that this passage deals solely with the conquest of the mainland city of old Tyre’.

[16] ‘The prophecy against Egypt does not seem to have come to pass in every detail either, but the book was probably completed and its authority established by the time this became clear.’, Thomas Renz, The Rhetorical Function of the Book of Ezekiel (Leiden: Brill, 2002), 98.

[17] J. B. Pritchard (ed.), Ancient Near Eastern Texts (3rd ed. with supplement. Princeton: Princeton UP, rev. 1969), 308.

[18] ‘That Nebuchadrezzar actually conquered Egypt is suggested by a number of very powerful pieces of evidence’, Emmet John Sweeney, The Ramessides, Medes, and Persians, Ages In Alignment Series, volume 4 (Algora Publishing, 2008), 153; ‘Nebuchadrezzar’ is the more accurate transliteration of the name ‘Nebuchadnezzar’.

[19] Ibid., p. 153.

[20] Ibid., p. 153.

[21] Ibid., p. 153.

[22] Jeremiah 43: 8 At Tahpanhes the LORD spoke to Jeremiah. 9 “Take some large stones and bury them in the mortar of the clay pavement at the entrance of Pharaoh’s residence here in Tahpanhes. Do it while the people of Judah present there are watching. 10 Then tell them, ‘The LORD God of Israel who rules over all says, “I will bring my servant King Nebuchadnezzar of Babylon. I will set his throne over these stones which I have buried. He will pitch his royal tent over them.

[23] ‘In short, the prophecy of Jeremiah that the king of Babylon would spread his royal pavilion at the entrance of the pharaoh’s house in Tahpanheth (Daphnae) was fulfilled.’, ibid., p. 153.

[24] Ibid., p. 153.

[25] ‘While scholars differ over the precise year, a date between 65 and 75 CE is accepted by a wide variety of scholars of very different ideological persuasions.’, James G. Crossley, The Date of Mark’s Gospel: Insight from the Law in Earliest Christianity (vol. 266; Journal for the Study of the New Testament Supplement Series; London; New York: T&T Clark International, 2004), 1.

[26] ‘This now becomes an argument of powerful collective weight for Mark to have been written before the late forties and if this is combined with the analysis of Mark 13 in Chapter 2 it is unlikely that it was written no earlier than the mid to late thirties.’, ibid., p. 208.

[27] Maurice Casey, Aramaic Sources of Mark’s Gospel (vol. 102;  Society for New Testament Studies; Cambridge University Press, 1998), 260.

[28] ‘According to Josephus,17 the Herodian temple was begun in 20/19 BCE; it was completed shortly before the war with Rome.18 If this scene may be used to date the events,19 it would have taken place on Passover of the year 28.’, Ernst Haenchen, Robert Walter Funk, and Ulrich Busse, John: a Commentary on the Gospel of John (Hermeneia—a Critical and Historical Commentary on the Bible; Philadelphia: Fortress Press, 1984), 184.

[29] ‘Work was still going on at his [Herod’s] death, and for that matter, for long after. The Temple was not completed until A.D. 63.’, Leon Morris, The Gospel According to John (The New International Commentary on the New Testament; Grand Rapids, MI: Wm. B. Eerdmans Publishing Co., 1995), 176.

h1

Living On The Edge: challenges to faith

September 1, 2013

Today Christians in the Western world are typically living in a post-Christian society. Christian beliefs are met with skepticism, and people see little reason to believe. Christians are confronted with daily challenges to their faith, and often struggle to understand the relevance of Christianity to modern life.

The book ‘Living On The Edge: challenges to faith‘ (due to be printed in November 2013), addresses those concerns. For an overview of the book, click here.

h1

Is the Bible’s chronology of the kings of Israel accurate?

July 30, 2011

The Challenge

In the late 19th century, critical scholar Julius Wellhausen claimed the Biblical chronology of the kings of Israel was a literary invention for religious purposes, which had been edited and revised several times from a variety of different sources, rather than a genuine historical record.[1]

For the next 70 years, critical scholars continued to treat the chronology as historically worthless and irreconcilable.[2]

The Facts

In 1951, Biblical scholar Edwin Thiele published ‘The Mysterious Numbers of the Hebrew Kings’, a harmonization of the Biblical record of the kings of Israel (originally as a doctoral dissertation). By the time of the second edition (slightly revised), it was recognized that Thiele’s work was a significant breakthrough in establishing the historical validity of the Biblical chronology.[3]

Reception

Though criticisms have been made of Thiele’s chronology,[4] [5] [6] [7] its value and general validity have been acknowledged widely.[8] [9] [10] [11] [12]

It remains the typical starting point for study of the chronology of the kings of Israel and Judah[13] [14] with few modifications,[15] [16]   and has been applied successfully in other fields of Ancient Near East study, such as the chronologies of Assyria and Babylon.[17]

The reliability of the chronologies in 1-2 Kings has been supported by archaeological evidence; Grabbe notes that the chronology in these books ‘agrees with what can be gleaned from extra-biblical sources’, and that ‘even if we had no external sources we could have reasonable confidence in the biblical sequence’.[18]


[1] ‘That a process of alteration and improvement of the chronology was busily carried on in later times, we see from the added synchronisms of the kings of Israel and Judah,’, Wellhausen, ‘Prolegomena to the History of Israel’, p. 278 (1885).

[2] ‘Driver remarked that, “the length of the reigns of the various kings is not the same according to the traditional and the synchronistic figures. Since, however, it is clear on various grounds that these synchronisms are not original, any attempt to base a chronological scheme on them may be disregarded.” Kittel stated his view that, “Wellhausen has shown, by convincing reasons, that the synchronisms within the Book of Kings cannot possibly rest on ancient tradition, but are on the contrary simply the products of artificial reckoning. . . The Israelitish numbers and the parallel numbers referring to Judah do not agree at the points at which we are able to compare them.” Robinson also was impressed by Wellhausen’s evaluation: “Wellhausen is surely right in believing that the synchronisms in Kings are worthless, being merely a late compilation from the actual figures given.” * R. H. Pfeiffer’s opinion was that, “The chronology based on the synchronisms is of course less reliable than the one based on the regnal periods, since the synchronisms were figured from the regnal periods. Neither chronology is wholly accurate . . . In spite of these discrepancies, inaccuracies, and errors, the chronology of Kings is not fantastic.” 5 J. F. McCurdy expressed himself to the effect that, “Many of the numbers given, especially the synchronisms, are erroneous, as is proved by the fact that no attempt to harmonize the two series has been successful . . . Startling inconsistencies are also found where the several synchronisms for the same king are worked out.” K. Marti gave his observation: “The synchronistic notes betray their character as ‘subjective additions of the Epitomator.’ It is clear, to begin with, that this noting of synchronisms was not in actual use during the existence of the two kingdoms. . . Almost along the whole line, the discrepancy between synchronisms and years of reign is incurable.” C. H. Gordon observed: “The numerical errors in the Books of Kings have defied every attempt to ungarble them. Those errors are largely the creation of the editors who set out to write a synchronistic history of Judah and Israel, using as sources two sets of unrelated court chronicles. Combining two elaborate sets of figures was not an easy task. But even with due regard for the difficulties involved, the editors did not execute the synchronisms skillfully.”’, Thiele, ‘Synchronisms of the Hebrew Kings – A Re-evaluation: I’, Andrews University Seminary Studies, pp. 14-125 (1), 1963.

[3]A marked advance in biblical scholarship was made in the publication of The Mysterious Numbers of the Hebrew Kings, U. of Chicago Press, 1951, by Dr. E. R. Thiele. In his revised edition in 1965 (Eerdmans Pub. Co., Grand Rapids, Mich.), Dr. Thiele asserts the soundness of his basic thesis and conclusions as confirmed by scholars since his first edition.’, editorial, Journal of the Evangelical Theological Society (9.1.60), 1966.

[4] ‘Thiele’s view contains positive elements, but it also poses numerous difficulties. He incorrectly understood the annals of Tiglath-pileser III, and his determination that Menahem died in 742 contradicts the testimonies of the contemporaneous Assyrian inscriptions.10 In his desire to resolve the discrepancies between the data in the Book of Kings, Thiele was forced to make improbable suppositions. He assumed that the system of counting the years of reign changed every few generations, or even after a few decades. This is improbable, and cannot be proved. Similarly, he presumes that the Northern and Southern Kingdoms numbered their years both by the local count and by that practiced by the other kingdom, also for short periods, while this practice ceased in other periods. Thiele even went so far as to assume that while this practice had fallen into disuse, there were scribes who continued to calculate the years in accordance with it. There is no basis for Thiele’s statement that his conjectures are correct because he succeeded in reconciling most of the data in the Book of Kings, since his assumptions regarding Biblical chronological principles are derived from the chronological data themselves, whose reliability is unclear.’, Galil, ‘The Chronology of the Kings of Israel and Judah’, p. 4 (1996).

[5]but his harmonizing approach has not gone unchallenged, especially because of the many shifts in the basis of reckoning dates that it requires (e.g., Jepsen 1968: 34–35)—shifts which were unlikely in actual practice. The numerous extrabiblical synchronisms he invokes do not always reflect the latest refinements in Assyriological research (cf. E.2.f below). In many cases, he posits an undocumented event in order to save a biblical datum (e.g., the circumstances surrounding the appointment of Jeroboam II as coregent; Thiele 1983: 109)’, Cogan, ‘Chronology (Hebrew Bible)’, in Freedman, (ed.), ’The Anchor Yale Bible Dictionary’, volume 1, p. 1006 (1996)’, Freedman, DN (1996).

[6]  ‘Despite that fact of scholarly dedication, neither Thiele’s carefully argued University of Chicago dissertation, nor anyone else’s, has achieved as yet universal acceptance.’, Kaiser, ‘A History of Israel: From the bronze age through the Jewish Wars’, p. 293 (1998).

[7]Not all scholars are convinced by this solution, and commentators on the prophetic books often accept that dates can only be approximate.’, McConville, ‘Exploring the Old Testament, Volume 4: The Prophets’, p. viii (2002).

[8] ‘The chronology most widely accepted today is one based on the meticulous study by Thiele. Wiseman, ‘1 and 2 Kings’, Tyndale Old Testament Commentaries, p. 27 (1993).

[9]Increasingly his chronological scheme has come to dominate the majority of scholarly works and it is unlikely that his system can ever be overthrown without altering some well-established dates in Near Eastern history, for Thiele’s chronology is now inextricably locked into the chronology of the Near East.’, McFall, ‘A Translation Guide to the Chronological Data in Kings and Chronicles’, Bibliotheca Sacra (148.589.42-43), 1996.

[10] ‘Thiele’s system of chronology has been well received over the past 40 years and is now accepted as the basis for Israel’s chronology in a growing number of standard scholarly works.’, ibid., p. 42; see for example: Mitchell, ‘Israel and Judah until the Revolt of Jehu (931-841 B.C.)’, Cambridge Ancient History, volume 3, part 1, p. 445 (1982); Finegan, ‘Handbook of Biblical Chronology’, p. 249 (rev. ed.1998); Hess, ‘Chronology (Old Testament)’, in Porter (ed.), ‘Dictionary of Biblical Criticism and Interpretation’, p. 55 (2007).

[11]Thiele’s chronology is fast becoming the consensus view among Old Testament scholars, if it has not already reached that point.’, McFall, ‘The Chronology of Saul and David’, Journal of the Evangelical Theological Society (53.101.215), 2010.

[12]Thiele’s chronology (which differs from that of the present paper in only a few places) won the respect of historians because its dates agree with the following dates in Assyrian and Babylonian history: the Battle of Qarqar in 853 bc; the tribute of Jehu to Shalmaneser III in 841 bc; the capture of Samaria by Shalmaneser V in 723 bc; the invasion of Sennacherib in 701 bc; the Battle of Carchemish in 605 bc; and the first capture of Jerusalem by Nebuchadnezzar in 597 bc.’, Young, ‘Tables of Reign Lengths From the Hebrew Court Recorders’, Journal of the Evangelical Theological Society (48.2.232), 2005.

[13] ‘Thiele’s work has become a cornerstone of much recent chronological discussion (cf. De Vries IDB 1: 580–99; IDBSup: 161–66);’, Cogan, ‘Chronology (Hebrew Bible)’, in Freedman, (ed.), ’The Anchor Yale Bible Dictionary’, volume 1, p. 1006 (1996)’, Freedman, DN (1996).

[14] ‘Although some would prefer to see transmission errors where Thiele invokes the above principles, his chronology remains the starting point for all discussions of the debate.’, Hess, ‘Chronology (Old Testament)’, in Porter (ed.), ‘Dictionary of Biblical Criticism and Interpretation’, p. 55 (2007).

[15] ‘After 40 years Thiele’s chronology has not been significantly altered or proved to be false in any major area except in the matter of Hezekiah’s coregency.’, McFall, ‘A Translation Guide to the Chronological Data in Kings and Chronicles’, Bibliotheca Sacra (148.589.42), 1996.

[16]It remained then for others to complete the application of principles that Thiele used elsewhere, thereby providing a chronology for the eighth-century kings of Judah that is in complete harmony with the reign lengths and synchronisms given in 2 Kings and 2 Chronicles. The most thorough work in this regard was Leslie McFall’s 1991 article in Bibliotheca Sacra.22 McFall made his way through the reign lengths and synchronisms of Kings and Chronicles, and using an exact notation that indicated whether the years were being measured according to Judah’s Tishri years or Israel’s Nisan years, he was able to produce a chronology for the divided monarchies that was consistent with all the scriptural texts chosen.’, ibid., pp. 105-106.

[17] ‘In a 1996 article, Kenneth Strand wrote, “What has generally not been given due notice is the effect that Thiele’s clarification of the Hebrew chronology of this period of history has had in furnishing a corrective for various dates in ancient Assyrian and Babylonian history.”28 The purpose of Strand’s article was to show that Thiele’s methodology accomplished more than just producing a coherent chronology from scriptural data. His chronology, once produced, proved useful in settling some troublesome problems in Assyrian and Babylonian history.’, Young, ‘Inductive And Deductive Methods As Applied To OT Chronology’, Master’s Seminary Journal (18.1.112-113), 2007.

[18] ‘Grabbe suggests that the names and sequence of kings in Israel and Judah, and their approximate chronological placement, agrees with what can be gleaned from extra-biblical sources. To this extent the biblical framework (meaning primarily 1 and 2 Kings) is reliable: even if we had no external sources we could have reasonable confidence in the biblical sequence of Jeroboam I, Nadab, Baasha, Elah, Omri, Ahab, Jehu, etc. in Samaria, and David, Solomon, Rehoboam, Abijam, Asa, Jehoshaphat, etc. in Jerusalem, along with their interrelationships. Beyond that it starts to get more and more tricky, with decreasing reliability in the biblical narrative as the detail increases (this is a general statement, and there are sometimes exceptions in specific instances).’, Grabbe, ‘Reflections on the Discussion’, Grabbe (ed.), ‘Ahab Agonistes: The Rise and Fall of the Omri Dynasty’, p. 337 (2007).

h1

Is Solomon’s wealth a literary fiction?

April 30, 2011

The Challenge

The Biblical account of Solomon’s wealth has been described as unrealistic, in standard critical commentaries.[1] Many scholars are sceptical, [2] [3] [4] though some express their doubts cautiously.[5] [6]

Ancient Uses of Gold

Ancient uses of gold for construction which are analogous to Solomon’s include the tomb of Tutankhamen,[7] extensive use of gold plating on buildings in the reign of Tuthmosis III,[8] massive gold use on buildings of the Egyptian New Kingdom era,[9] and the same kind of gold usage in Babylonia and Assyria.[10] Millard also points out that items described as ‘of gold’ were not always solid gold; often they were covered in gold plate or gold leaf.[11]

Solomon’s Income

The Bible identifies ‘Ophir’ as one source of Solomon’s gold.[12] Although the location of Ophir is unknown, archaeological evidence identifies it as a source of gold.[13] Solomon’s income of 666 talents of gold in one year[14] [15]  is considered fictional by some commentators.[16]

Although this income is unique in Ancient Near East records,[17] the 120 talents of gold received by Solomon from Tyre[18] is matched and exceeded by gifts and tribute of gold from other Ancient Near East monarchs.[19] [20] [21] [22] [23]

The vast gold expenditure of pharaoh Orsokon I exceeded even Solomon’s, [24] and it is likely his wealth was the result of his father Sheshonq’s conquest of Solmon’s son Rehoboam.[25] [26] [27] [28]


[1]The gilding of the furnishings, as of the altar, is reasonable, but not that of the whole interior; cf. Stade, and Nowack, Arch., 2, 29, n. I.’, Montgomery, ‘A Critical and Exegetical Commentary on the Book of Kings’, p. 152 (1951).

[2]Such extravagant description appears to be a step forward in the process of exuberant imagination, continued by the Chronicler, for whose fancy even the 120-cubit high portico was overlaid with fine gold (2 Ch. 34ff.).’, ibid., p. 152.

[3] ‘Some have regarded this description as exaggerated.’, Hicks, ‘1 & 2 Chronicles’, College Press NIV Commentary, p. 306 (2001).

[4] ‘Some have questioned the authenticity of this description, labeling it unabashed exaggeration.’, Long, ‘1 & 2 Kings’, College Press NIV Commentary, p. 147 (2002).

[5]Despite all exaggerated accounts of Solomon’s wealth and commercial success, which were written to give him honor and prestige, there is an historical kernel in the reports of his wealth.’, Esler, ‘Ancient Israel: The Old Testament in its social context’, p. 105 (2006).

[6] ‘Evidently, we can not take the figures about Solomon’s mercantile activities and revenues given in the account at face value. They must have been fabulously exaggerated. Nevertheless, in Ishida’s assessment, which I share, “We can hardly deny the substantial historicity comprised in them” (p. 109).’, Corral, ‘Ezekiel’s Oracles Against Tyre: historical reality and motivations’, p. 112 (2002).

[7] ‘Here were many articles of furniture plated with sheets of gold, beaten and engraved, a wealth of elaborate golden jewellery, a golden dagger, the king’s gold mask, and, eclipsing all, his coffin of solid gold.7 Its weight is 110.4 kg (243 lbs). Particularly relevant for the present study are the shrines that stood in the tomb. There is a small wooden shrine (50 cms high, 26.6 cms wide, 32 cms deep, 19¾ x 10½ x 12¾ inches) made to hold a statue. Sheets of gold cover it entirely, within and without, embossed and engraved with scenes of the king’s life, magical figures, and inscriptions.’, Millard, ‘Solomon In All His Glory’, Bible and Spade (11.2-3-4.64-65), 1982.

[8] ‘In the Temple of the Sacred Boat at Karnak stood twelve columns erected by Tuthmosis III, about 1450 BC, each about 3½ metres high, designed to represent bundles of papyrus. Each was entirely covered with gold, fastened in slits cut at suitable points in the pattern. In another hall at Karnak were fourteen columns. Their design was similar, a papyrus stem, and they, too, were plated with gold from top to bottom. These pillars were larger; an inscription states that they were 31 cubits, that is 16.25 metres high (53 feet).’, ibid., p. 67.

[9] ‘Tuthmosis III (c. 1490–1436 BC) recorded his building of a shrine ‘plated with gold and silver’, and of a floor similarly made. Amenophis III in the next century decorated several structures in this way. Of one temple in honour of Amun at Thebes he claimed it was ‘plated with gold throughout, its floor is adorned with silver, all its portals with electrum’, while the temple at Soleb had the same treatment, except that ‘all its portals are of gold’. Ramesses II (c. 1297–1213 BC) provided his mortuary temple at Abydos with doors ‘mounted with copper and gilded with electrum’. Later in this period, Ramesses III (c. 1183–1152 BC) ornamented temples in exactly the same way. At Medinet Habu he constructed a shrine of gold with a pavement of silver, and doorposts of fine gold.’, ibid., p. 68.

[10] ‘Esarhaddon of Assyria (680–669 BC) told how he restored the shrine of his national god, Assur, and ‘coated the walls with gold as if with plaster’. His son Ashurbanipal claimed much the same, ‘I clad its walls with gold and silver’. In Babylon a century later Nebuchadnezzar recorded his enrichment of the shrines of his gods, ‘I clad (them) in gold, and made them bright as day’, and Nabonidus (555–539 BC) followed him, ‘I clad its walls with gold and silver, and made them shine like the sun’. The tradition stemmed from much earlier times in Babylonia, for Entemena of Lagash built a temple for his god ‘and covered it with gold and silver’ about 2400 BC.16’, ibid., pp. 68-69.

[11]While words like ‘a gold statue’ or ‘a gold bed’ in ancient documents should not be pressed to mean ‘made of solid gold throughout’ or ‘the purest gold’, they can be understood to mean ‘gold all over’, that is to say, nothing else could be seen.’, ibid., pp. 69-70.

[12] 1 Kings 9: 28 They sailed to Ophir, took from there four hundred twenty talents of gold, and then brought them to King Solomon.

[13]The expression “gold of Ophir” occurs not only in the Bible, but also on an eighth-century B.C. ostracon* found at Tell Qasile in Israel. That ostracon, while showing that the name was current to designate the origin or type of gold, throws no light on Ophir’s location.’, Millard, ‘Does the Bible Exaggerate King Solomon’s Golden Wealth?’, Biblical Archaeology Review (15.03), May/June 1989.

[14] 1 Kings 10:14 Solomon received 666 talents of gold per year; commentators are divided as to whether this represents an annual income, or the income of one particular year.

[15] ‘On the basis of these figures, Solomon’s gold can be computed as: 120 talents==3,960 kg==3.9 tons from Tyre, and the same from Sheba; 420 talents==13,860 kg==13.6 tons from Ophir; 666 talents==21,978 kg==21.6 tons in one year.’, Millard, ‘Solomon In All His Glory’, Bible and Spade (11.2-3-4.72), 1982.

[16] ‘Indeed, J.B. Pritchard argues that the narrative’s references to gold, pure gold and silver and its allusions to the respect which Solomon’s peers showed to him are ‘popular—even folkloristic’ elements of the history of Solomon’s age.’, Younger Jr, ‘The Figurative Aspect and the Contextual Method in the Evaluation of the Solomonic Empire: 1 Kings 1–11’, in Clines, Fowl & Porter (eds.), ‘The Bible in Three Dimensions: Essays in Celebration of Forty Years of Biblical Studies in the University of Sheffield’, p. 159 (1990).

[17]The only ancient text that reports the annual income of a powerful king in Old Testament times is the Hebrew Bible. In 1 Kings 10:14 the figure of 666 talents of gold (almost 25 U.S. tons) is given for Solomon. This may refer to a particular year, just as the 420 talents (15.75 U.S. tons) from Ophir refers to a particular source (1 Kings 10:11). Only two figures in ancient records approach the amount of 666 talents: the total of Pharaoh Osorkon’s gift to the gods and the amounts of treasure Alexander the Great found in Persia.’, Millard, ‘Does the Bible Exaggerate King Solomon’s Golden Wealth?’, Biblical Archaeology Review (15.03), May/June 1989.

[18] 1 Kings 9: 28 They sailed to Ophir, took from there four hundred twenty talents of gold, and then brought them to King Solomon.

[19] ‘We learn from firsthand sources that Metten II of Tyre (ca. 730) paid a tribute of 150 talents of gold to our old acquaintance Tiglath-pileser III of Assyria, while in turn his successor Sargon II (727-705) bestowed 154 talents of gold upon the Babylonian gods – about 6 tons in each case. Going back almost eight centuries, Tuthmosis III of Egypt presented about 13.5 tons (well over 200 talents) of gold in nuggets and rings to the god Amun in Thebes, plus an unknown amount more in a splendid array of gold vessels and cult implements. Worth almost a third of Solomon’s reputed annual gold revenue, this was on  just one occasion, to just one temple.’, Kitchen, ‘On the Reliability of the Old Testament’, pp. 133-134 (2003).

[20] ‘So a king of Assyria wrote to the Pharaoh about 1350 BC, ‘Gold is like dust in your land, one simply gathers it up.’ A contemporary king repeated this statement six times in letters to the Pharaoh! The Assyrian went on ‘Why do you think it is so valuable? I am building a new palace, send me enough gold to decorate it properly! When my ancestor wrote to Egypt, he was sent twenty talents of gold. . . . When (another king) wrote to Egypt to your father, he sent him twenty talents of gold . . . send me much gold!’ (Twenty talents by Babylonian standards was 600 kg or 11.7 cwts.).’, Millard, ‘Solomon In All His Glory’, Bible and Spade (11.2-3-4.73), 1982.

[21] ‘When Damascus surrendered to Adadnirari III, probably in 796 BC, the Assyrian received 2,300 talents of silver (69,000 kg; 67.76 tons), 20 talents of gold (600 kg; 1,320 lbs), and much else. Some sixty years later Tiglath-pileser III subjugated Samaria, placing Hoshea on the throne as his nominee. Samaria paid 10 talents of gold (300 kg; 660 lbs) as tribute (and an unknown amount of silver). The same emperor received the submission of Tyre, and with it the large sum of 150 talents of gold (4,500 kg; 4.4 tons).’, ibid., p. 74.

[22] ‘During the reign of Tuthmosis III the yield of the gold fields at Wawat in Nubia (the Sudan) for three years was 232.4 kg (512 lbs), 258.8 kg (570 lbs), and 286.1 kg (630 lbs). These may be exceptional figures, yet they show what sort of income was available from a single source. In the Annals of the same pharaoh, the booty taken between his twenty-second and his forty-second years amounted to over 11,500 kg (11.3 tons) of gold. His successor, Amenophis II (c. 1427–1401 BC) claimed the weight of gold vessels he took from the Levant was 6,800 deben (618.5 kg; 1,360 lbs).’, ibid., p. 75.

[23] ‘None of these figures approach the amounts recorded for Solomon except for the booty gathered by Tuthmosis III (11,500 kg; 11.3 tons).’, ibid., p. 75.

[24] ‘In Egypt Shishak’s successor Osorkon I gifted some 383 tons of gold and silver to the gods and temples of Egypt in the first four years of his reign, many of the detailed amounts being listed in a long inscription (now damaged) (figs. 22A, B). That sum would (in weight) be equivalent to almost seventeen years of Solomon’s annual gold revenue,’, Kitchen, ‘On the Reliability of the Old Testament’, p. 134 (2003).

[25] 1 Kings 1425 In King Rehoboam’s fifth year, King Shishak [Sheshonq] of Egypt attacked Jerusalem. 26 He took away the treasures of the LORD’s temple and of the royal palace; he took everything, including all the golden shields that Solomon had made.

[26]His reign is poorly documented, nothing hints at a far-reaching military adventure, bringing home rich booty.’, Millard, ‘Solomon In All His Glory’, Bible and Spade (11.2-3-4.76), 1982.

[27] ‘Osorkon’s father was Sheshonq 1 (c. 945–924 BC), the Shishak who took the gold from Solomon’s Temple in Jerusalem and from the Judaean treasury.’, ibid., p. 76.

[28] ‘Where could Osorkon have obtained such immense wealth, to spend on such a scale after only three and a third years of his reign? Barely five years earlier, Osorkon’s father Shishak had looted the wealth of Jerusalem. It seems unlikely to be a mere coincidence that almost immediately after that event Osorkon could dispose so freely of so much gold and silver.*The vast amounts of Solomon’s golden wealth may have ended up, at least in part, as Osorkon’s gift to the gods and goddesses of Egypt.’, Kitchen, ‘Where Did Solomon’s Gold Go?’, Biblical Archaeology Review (15.03), May/June 1989.

h1

What is archaeological ‘minimalism’?

April 26, 2011

Definition

The ‘minimalist’ view is that archaeology provides little or no support for the Biblical history.[1] [2] The best known adherents are Philip Davies,[3]  Lester Grabbe,[4] Niels Lemche,[5] Thomas Thompson,[6] and Keith Whitelam.[7] However, Israel Finkelstein and Neil Silberman are the only two prominent archaeologists associated with minimalist views.

Kenneth Kitchen

Kitchen[8] has raised numerous objections to minimalist claims, rejecting Thompson’s assertion that the Hebrew Tabernacle is a literary fiction,[9] that the Merneptah Stele is not reliable evidence for a people named ‘Israel’ in early 13th century Canaan,[10] that the Tel Dan Stele does not refer to a Hebrew ‘House of David’,[11] that the description of Solomon’s wealth is legendary,[12] and that the use of the first person perspective in the Mesha Stele indicates a post-mortem or legendary account.[13] [14] [15] Kitchen has also criticized Finkelstein and Silberman.[16] [17]

William Dever

Though far more sceptical than Kitchen, Dever[18] has nevertheless opposed minimalism vigorously. [19] [20] [21] [22]

Israel Finkelstein

Despite sympathies with some minimalist views, Finkelstein has rejected strongly the minimalist claims concerning Persian era Hebrew scribes,[23] that the ‘lists and details of royal administrative organization in the kingdom of Judah’ are fictional,[24] and that the Hebrew King David never existed.[25] [26] He has also acknowledged strong archaeological support for certain parts of the Biblical record.[27]

Amihai Mazar

With more in common with Finkelstein than the minimalists,[28] Mazar[29]  takes a moderate though critical view of the Biblical history.[30] [31]


[1] The ‘maximalist’ view is that archaeology overwhelmingly supports the Biblical history, and the moderate view is that archaeology substantially supports the Biblical history but that not all of the history can be supported directly from archaeology.

[2] ‘A recent trend in Syro-Palestinian archaeological study known as minimalism or revisionism suggests that Israel was created in the Hellenistic period.’, McCarty & Merrill, ‘Review: What Did the Biblical Writers Know and When Did They Know It? By William G. Dever.’, Bibliotheca Sacra (161.641.115), 2004.

[3] Biblical scholar.

[4] Historian.

[5] Biblical scholar.

[6] Biblical scholar.

[7] Biblical scholar.

[8] Egyptologist, Assyriologist, and archaeologist.

[9] ‘In so doing he ignores the whole of the comparative data that show clearly that the tabernacle was a product of Egyptian technology from the overall period 3000 to 1000 D.C. (plus Se-mitic analogues, 1900-1100), and would be unable to account for such facts.’, Kitchen, ‘On the Reliability of the Old Testament’, pp. 450-451 (2005).

[10] ‘The Israel of Merenptah’s stela was, by its perfectly dear determinative, a people (= tribal) grouping, not a territory or city-state; rare statements to the contrary are perverse nonsense, especially given the very high level of scribal accuracy shown by this particular monument.’, ibid., p. 451.

[11] ‘(i) The name “David” may be unusual, but is not unparalleled. Long centuries before, it was borne by a West Semitic chief carpenter in about 1730 B.C. on an Egyptian stela formerly in the collection at Rio de Janeiro. (ii) Dwd is neither the name (which Thompson admits) nor an epithet of a deity. Others are beloved of deities (for which references are legion!), but male deities are not beloved of others, human or divine (only goddesses are beloved of their divine husbands in Egypt). (iii) Mesha’s stela is ninth, not eighth, century. (iv) On Mesha’s stela dwd(h) is not a divine epithet of YHWH or anyone else. (v) Contrary to TLT, “House of X” does  mean a dynastic founder, all over the Near East, in the first half of the first millennium B.C.; it was an Aramean usage that passed into Assyrian nomenclature, and examples are common. (vi) Again, the expression, in part of its usage, is like the British “House of Windsor”, etc. Such usages were not peculiar to Aram, Assyria, and Judah either: in Egypt, the official title given to the Twelfth Dynasty (Turin Canon) was “Kings of the House (lit. ‘Residence’) of Ithet-Tawy” = ‘the Dynasty of Ithet-Tawy”. And the Thirteenth Dynasty was duly entitled “Kings who came after the [House of] King Sehetepibre” (founder of the Twelfth Dynasty). (vii) The charge of forgery is a baseless slur against the Dan expedition, without a particle of foundation in fact.’, ibid., pp. 452-453.

[12] ‘The point of the comparisons drawn with external (and firsthand!) sources was precisely that Solomon’s wealth (even as stated in Kings) was not exceptional or “fabulous/legendary” in its wider context. He was a pauper compared with (e.g.) Osorkon I, who, less than a decade after Solomon’s death, spent sums that massively outstrip Solomon’s stated income, and gave detailed accounts. The layering that TLT objects to was customary. At Karnak in Egypt, some temple columns were grooved to fit sheet gold from top to bottom, not mere “plastering.” As a touch of throwaway wealth, one need look no further than the recently discovered burials of two Assyrian queens. Solomon had just one golden throne? One pharaoh was sent ten at a time!’, ibid., p. 454.

[13] ‘Use of the first person by a monarch does not belong exclusively to either postmortem memorial texts or to later legends about such kings. A huge army of texts shows up the falsity of his presumption.’, ibid., p. 456.

[14]For first person, not postmortem, see (e.g.) Lipit-Ishtar (247), Warad-Sin/Kudur-mabuk (251-52), Rim-Sin I (253, his third text), Hammurabi (256-57), Ammi-ditana (258-59), and Shamshi-Adad I (259), all early second millennium. In the first millennium every major Assyrian king did exactly likewise, in various editions of their annals that were anything but postmortem, from Tiglath-pileser I to Assurbanipal (cf. ANET, 274-301; CoS II,261-306; RIMA, 1-3 passim).’, ibid., p. 456.

[15]Mesha’s stela is a contemporary building plus victory text, exactly like so many other inscriptions in the ancient Near East in all places and periods; the first-person formulation is irrelevant to its status, as the foregoing examples (far from exhaustive!) show.’, ibid., p. 457.

[16] ‘His reevaluation of the realm of Omri and Ahab is refreshing but wildly exaggerated, especially in archaeological terms. As others have shown amply, the redating will not work (cf. chap. 4, sec. 3 above).’, ibid., p. 464.

[17]The Philistines of Gerar (not those of the Pentapolis!) are a very different lot from the Iron Age group of that name. The term is a probably twelfth-century one substituted for Caphtorim or the like, precisely as Dan was substituted for Laish in Gen. 14:]4.’, ibid., p. 465.

[18] Archaeologist.

[19] ‘There are some who claim that the Bible contains little or no historical information about ancient Israel. I want to combat these “minimalist” or “revisionist” views of the history of ancient Israel by showing how archaeology can and does illuminate a historical Israel in the Iron Age of ancient Palestine (roughly 1200–600 B.C.E.)’, Dever, ‘Save Us from Postmodern Malarkey’, Biblical Archaeology Review (26.02), March/April 2000.

[20]Davies does not even cite the standard handbook, Mazar’s Archaeology of the Land of the Bible,’, Dever, ‘Who Were the Early Israelites? And Where Did they Come From?’, p. 138 (2003).

[21] ‘That same year Whitelam wrote an article for the Sheffield Journal for the Study of the Old Testament on “the realignment and transformation of Late-Bronze-Iron Age Palestine.” It was so full of caricatures of modern archaeological theory and results that I felt compelled to answer it in the same journal.’, ibid., p 139.

[22] ‘Thus he [Thompson] published two years later his revisionist treatment of ancient Israel: The Mythic Past: Biblical Archaeology and the Myth of Israel. Despite its subtitle, this work has next to nothing to do with real archaeology.’, ibid., p. 141.

[23] ‘First of all, as the biblical scholar William Schniedewind has indicated, literacy and extensive scribal activity in Jerusalem in the Persian and early-Hellenistic periods were much less influential than in the seventh century b.c.e. The assumption is inconceivable that in the fifth, or fourth, or even second centuries b.c.e., the scribes of a small, out-of-the-way temple town in the Judean mountains authored an extraordinarily long and detailed composition about the history, personalities, and events of an imaginary Iron Age “Israel” without using ancient sources.’, Finkelstein, ‘Digging for the Truth: Archaeology and the Bible’, in Schmidt (ed.), ‘The Quest for the Historical Israel’, Archaeology and Biblical Studies, number 17, p. 13 (2007).

[24] ‘The sheer number of name lists and details of royal administrative organization in the kingdom of Judah that are included in the Deuteronomistic History seems unnecessary for a purely mythic history. In any event, if they are all contrived or artificial, their coincidence with earlier realities is amazing.’, ibid., p. 13.

[25] ‘This argument suffered a major blow when the Tel Dan basalt stele was discovered in the mid-1990s.’, ibid., p. 14.

[26] ‘This was the first time that the name “David” was found in any contemporary source outside the Bible, in this case only about a century after his own supposed lifetime.’, ibid., p. 14.

[27] ‘Archaeological excavations and surveys have confirmed that many of the Bible’s geographical listings—for example, of the boundaries of the tribes and the districts of the kingdom—closely match settlement patterns and historical realities in the eighth and seventh centuries b.c.e. Equally important, the biblical scholar Baruch Halpern showed that a relatively large number of extra-biblical historical records—mainly Assyrian—verify ninth- to seventh-century b.c.e. events described in the Bible: the mention of Omri in the Mesha stele, those of Ahab and Jehu in the Shalmaneser III inscriptions, Hezekiah in the inscriptions of Sennacherib, Manasseh in the records of Esarhaddon and Ashurbanipal, and so on. No less significant is the fact, as indicated by the linguist, Avi Hurwitz, that much of the Deuteronomistic History is written in late-monarchic Hebrew, which is different from the Hebrew of post-exilic times.’, ibid., pp. 13-14.

[28]‘ Our views differ on certain important issues, but we share more in common than we do with either of the two extreme groups described above.’, Mazar, ‘On Archaeology, Biblical History, and Biblical Archaeology’, in Schmidt (ed.), ‘The Quest for the Historical Israel’, Archaeology and Biblical Studies, number 17, p. 29 (2007).

[29] Archaeologist.

[30] ‘My own choice is to follow those who claim that the initial writing of the Torah (the Pentateuch or Tetrateuch), of the Deuteronomistic History and large parts of the prophetic and wisdom literature took place during the late monarchy (eighth to early-sixth centuries b.c.e.), while during the exilic and post-exilic periods they underwent further stages of editing, expansion, and change. Yet, I also accept the

view of many scholars that the late-monarchic authors utilized earlier materials and sources.’, ibid., p. 29.

[31] ‘Both Assyrian inscriptions and local inscriptions like the stelae of Mesha, king of Moab, and of Hazael, king of Damascus (better known as the Tel Dan inscription), confirm that the general historical framework of the Deuteronomistic narrative relating to the ninth century was based on reliable knowledge of the historical outline of that century. Our understanding of the periods preceding the ninth century is of course foggier.’, ibid., p. 30.

h1

Does the archaeological ‘Low Chronology’ disprove the Biblical narrative?

April 23, 2011

The Challenge

The ‘Low Chronology’ is a proposed redating of the Iron Age,[1] dating the reigns of David and Solomon to a time during which there is no archaeological evidence supporting them.[2]

The Objections

Proposed at least as early as the 1980s,[3]  the redating received almost no support,[4] and was resisted strongly by the archaeological consensus.[5]

Objections were raised by archaeologists including Dever (1997),[6] Mazar (1997, 1999),[7] Zarzeki-Peleg (1997),[8] Ben-Tor (1998), and Ben Ami (1998).[9]

Finkelstein responded, but criticism was renewed in 2000 by Na’aman[10] and Ben-Tor.[11] Over the next five years Finkelstein was virtually the only promoter of the theory. [12] [13] [14]  [15] [16] [17] [18]

The Evidence

Mazar and Dever note evidence agreeing with the Bible’s description of Jerusalem under David and Solomon.[19]  [20] Garfinkel likewise says evidence supports the description of the Israelite battles with the Philistines.[21]

Architecture at Khirbet Qeiyafa indicates David ruled an established state, as in the Biblical narrative.[22] Carbon 14 dated olive pits at the site have an age within the traditional date for the reign of David.[23]

Dead and Buried

Garfinkel believes the evidence from Khirbet Qeiyafa to be conclusive, [24] [25] and has declared ‘Low chronology is now officially dead and buried’.[26]


[1] ‘Proponents of the low chronology suggest that the end of the Iron Age I and the Iron Age IIA should be dated some eighty to one hundred years lower than the traditional chronology.’, Lehman, ‘The United Monarchy in the Countryside: Jerusalem, Judah and the Shephelah during the Tenth Century B.C.E’, in Vaughn & Killebrew (eds.), ‘Jerusalem in Bible and archaeology: The First Temple Period’, pp. 119-120 (2003; Iron Age I and Iron Age IIA are specific eras within the Iron Age.

[2]This suggested “Low Chronology” supposedly supports the replacement of this paradigm by a new one (in fact, similar to one presented earlier by David Jamieson Drake and others), according to which the kingdom of David and Solomon either did not exist or comprised at best a small local entity.’, Mazar, ‘The Search for David and Solomon: An Archaeological Perspective’, in Schmidt (ed.), ‘The Quest For the Historical Israel: debating archaeology and the history of Early Israel’, p. 119 (2007).

[3]“Revisionism” began on the archaeological front in the early 1980s, when several archaeologists o the Tel Aviv University set out to lower the conventional 10th century date of the distinctive four-entryway city gates and casement (or double) walls at Hazor, Megiddo, and Gezer to the early-mid-9th century BCE.’, Dever, ‘Biblical and Syro-Palestinian Archaeology’, in Perdue (ed.), ‘The Blackwell companion to the Hebrew Bible’, p. 137 (2001).

[4] ‘The Tel Aviv group’s idiosyncratic “low chronology,” however, was not accepted by the Jerusalem school, or by any European or American archaeologist (it still is not widely accepted, even by all Tel Aviv archaeologists).’, ibid., pp. 137-138.

[5] ‘The underlying premises of the Low Chronology were quickly challenged.’, Ortiz, ‘Deconstructing and Reconstructing the United Monarch’ , in Hoffmeier & Millard (eds.), ‘The Future of Biblical Archaeology: Reassessing Methodologies and Assumptions’, p. 128 (2004).

[6] ‘And I can tell you that not a single one of the other Israeli archaeologists agrees with this low chronology, except Israel Finkelstein.’ Dever, quoted in Shanks, ‘Face to Face: Biblical Minimalists Meet Their Challengers’, Biblical Archaeology Review (23.04), July/August 1997.

[7] ‘Mazar concluded that Finkelstein’s suggestion to push the date of the Philistine Monochrome pottery beyond the end of the Egyptian presence in Canaan is based on a debatable assumption (Tenet #2).’, ibid., p. 128.

[8] ‘A second article criticizing the Low Chronology was published by Anabel Zarzeki-Peleg.17 She also focused on the Iron Age stratigraphy of northern assemblages. Zarzeki-Peleg presented a ceramic typological study of three important northern sites (Megiddo, Jokneam, and Hazor) and concluded that the stratigraphical redating of the Low Chronology is not possible.’, ibid., p. 128.

[9] ‘The most significant studies, all opposed to Finkelstein’s “low chronology,” are those of Zarzeki-Peleg, 1997; Ben-Tor and Ben-Ami, 1998; and Mazar, 19991.’, Dever, ‘Biblical and Syro-Palestinian Archaeology’, in Perdue (ed.), ‘The Blackwell companion to the Hebrew Bible’, p. 202 (2001).

[10] ‘A second set of responses to Finkelstein’s Low Chronology came in a 2000 issue of BASOR.22 First, Nadav Na’aman challenged Finkelstein’s redating of the Philistine Monochrome pottery using Trojan Grey Ware from Lachish and Tel Miqne-Ekron.’, ibid, p. 129.

[11]‘A second article by Ben-Tor addressed Finkelstein’s redating of the northern sites, particularly Hazor.’, ibid., p. 129.

[12]‘In the meantime, his views are opposed by such leading archaeologists as Amihai Mazar of Hebrew University, excavator of Tel Rehov;* Amnon Ben-Tor of Hebrew University, excavator of Hazor;* Lawrence Stager of Harvard University, excavator of Ashkelon; and William Dever of the University of Arizona, excavator of Gezer. More to the point, Finkelstein’s low chronology has not been accepted even by his codirector at Megiddo, David Ussishkin. Ussishkin tells us that “on archaeological grounds it is quite possible (though not necessary) that some or all of [the structures in Stratum VA-IVB] originate in the 10th century B.C.E., during Solomon’s reign,” which is what the traditional chronology holds.’, Shanks, ‘Reviews:  Megiddo III—The 1992–1996 Seasons, Israel Finkelstein, David Ussishkin and Baruch Halpern, Editors’, Biblical Archaeology Review (6.06), November/December 2000.

[13] ‘What they do not tell the reader is that Finkelstein does not deny an Israelite state, but only down-dates its origins somewhat; and that his idiosyncratic “low chronology” is scarcely accepted by any other archaeologist.’, Dever, ‘What Did the Biblical Writers Know and When Did They Know it?’, p. 43 (2002).

[14] ‘It should not go unnoticed that not a single other ranking Syro-Palestinian archaeologist in the world has come out in print in support of Finkelstein’s ‘low chronology’.’, Dever, ‘Histories and Non-Histories of Ancient Israel: The Question of the United Monarchy’, in Day (ed.), ‘In Search of Pre-exilic Israel: proceedings of the Oxford Old Testament Seminar’, p. 73 (2003).

[15]‘The overwhelming consensus is, now more than ever, against Finkelstein’s low chronology, and therefore against his ‘new vision’ of ancient Israel.’, Dever, in Tel Aviv, volumes 30-31, p. 278 (2003).

[16]‘Demolishing Finkelstein’s supposed late date for the appearance of Philistine Bichrome pottery, based on an argument entirely from silence, leaves him without a leg to stand on for the remainder of his Iron I ‘low chronology’. While he continues to present it as fact, even claiming a growing consensus, there is not a shred of empirical (that is, stratigraphic) evidence to support this chronology.’, Dever, ‘Histories and Non-Histories of Ancient Israel: The Question of the United Monarchy’, in Day (ed.), ‘In Search of Pre-exilic Israel: proceedings of the Oxford Old Testament Seminar’, p. 73 (2003).

[17]‘Most senior archaeologists reject Finkelstein’s low chronology.’, Shanks, ‘Radiocarbon Dating: How to Find Your True Love’, Biblical Archaeology Review (31.01), January/February 2005; he cites ‘Amihai Mazar, Ephraim Stern, Amnon Ben-Tor, all of Hebrew University; Lawrence Stager of Harvard; William Dever and Seymour Gitin, the former and present directors of the Albright Institute in Jerusalem; and even Baruch Halpern, co-director with Finkelstein and David Ussishkin of the current excavation of Megiddo.’, but adds ‘But at this level of scholarship, you don’t simply count noses; you reason and argue! Recently, two brilliant younger archaeologists working at what is becoming a key site in the debate (Tel Dor on the Mediterranean coast) have parted company on this issue from their mentor, Hebrew University archaeologist Ephraim Stern, and now support Finkelstein’s low chronology.† Are the sands shifting?’.

[18] ‘Currently, Finkelstein is the only outspoken proponent of the Low Chronology.’, Ortiz, ‘Deconstructing and Reconstructing the United Monarch’ , in Hoffmeier & Millard (eds.), ‘The Future of Biblical Archaeology: Reassessing Methodologies and Assumptions’, p. 128 (2004).

[19] ‘Jerusalem of the tenth century B.C.E. is described by Finkelstein as a small and unimportant village. However, the “Stepped Stone Structure” in Area G in the City of David is a huge retaining wall that must have supported one of the largest buildings (perhaps the largest) of the 12th-10th centuries B.C.E. in the entire land of Israel. The pottery evidence indicates that it was founded during the Iron Age I (12th-11th centuries B.C.E.) and went out of use at some time after the tenth century. This fits the Biblical description of “The Citadel of Zion” (Metsudat Zion) as a Jebusite citadel captured by David and used as his stronghold (2 Samuel 5:7). In addition, Iron IIA pottery was found in almost every excavation area in the City of David. Jerusalem may not have been an enormous city during that time, but it definitely was much more than merely a small village, as Finkelstein contends. Outside of Jerusalem, monumental structures at Hazor, Megiddo and Gezer can, in my opinion, be dated to the tenth century B.C.E. Thus Yigael Yadin was probably correct in suggesting that these should be associated with Solomon’s building projects mentioned in 1 Kings 9:15.’, Mazar, ‘Does Amihai Mazar Agree with Finkelstein’s “Low Chronology”?’, Biblical Archaeology Review (29.02), March/April 2003.

[20] ‘If the biblical Solomon had not constructed the Gezer gate and city walls, then we would have to invent a similar king by another name.’, Dever, ‘What Did the Biblical Writers Know and When Did They Know it?’, p. 133 (2002).

[21]The geopolitical circumstances in the Elah Valley during the late 11th–early 10th centuries are quite clear. The mighty Philistine city state of Gath, ca. 30 hectares in area, was located only 12 km downstream from Khirbet Qeiyafa. This was a hostile border area, where the Kingdoms of Gath and Jerusalem had constant millenary conflicts. The story of David and Goliath is just one of many such “warrior tales” listed in 2 Sam 21:15–22 and 1 Ch 11:11–27. Even if many of these traditions are folkloristic in character, their chronology and geography bear historical memories. As by the end of the 9th century BCE Gath disappeared as a political power, these traditions must have been created at an earlier time.’, Garfinkel, ‘Khirbet Qeiyafa: Sha’arayim’, Journal of Hebrew Scriptures (8.22.6). 2008.

[22]Khirbet Qeiyafa is surrounded by a massive casemate city wall, 700 m long and 4 m wide. It is constructed of megalithic stones, quite often reaching a weight of 4–5 tons apiece, and in the eastern gate, even ca. 10 tons each. Our calculation suggests that 200,000 tons of stone were required for the construction of these fortifications. A four-chambered gate, its upper part constructed of ashlars, was located and excavated in the western part of the city. It is clearly a fortified town rather than a rural settlement.’, Garfinkel, ‘Khirbet Qeiyafa: Sha’arayim’, Journal of Hebrew Scriptures (8.22.5). 2008.

[23] ‘As Khirbet Qeiyafa is an Iron Age IIA site, we are left with a dating post-1000 BCE, that is, 1000–975 BCE (59.6%) or 1000–969 BCE (77.8%). These dates fit the time of King David (ca. 1000–965 BCE) and are too early for King Solomon (ca. 965–930 BCE).’, ibid., p. 3.

[24] ‘The four new C14 results from Khirbet Qeiyafa clearly indicate that the “low chronology” and the “ultra-low chronology” are unacceptable.’, ibid., p. 4-5.

[25] ‘The biblical text, the single-phase city at Khirbet Qeiyafa, and the radiometric dates each stands alone as significant evidence clearly indicating that the biblical tradition does bear authentic geographical memories from the 10th century BCE Elah Valley. There is no ground for the assumption that these traditions were fabricated in the late 7th century BCE or in the Hellenistic period.’, ibid., pp. 5-6.

[26] Garfinkel & Ganor, ‘Khirbet Qeiyafa: An Early Iron IIa Fortified City in Judah’, presentation to the American Schools of Oriental Research, slide 24 (2010).

h1

The Historicity of the book of Acts (5)

April 22, 2011

Paul’s Commission: Acts 9:1-2

It has been claimed there is no historical basis for Paul’s commission from the High Priest to extradite from Damascus to Jerusalem any Jews who had become Christians,[1] and that neither the High Priest nor the Sanhedrin had any jurisdiction in Damascus.[2] [3] [4]

Evidence & Commentary

Peerbolte raises a parallel in the history of the Maccabees, in which a Roman consul ordered Jewish rebels in Egypt to be extradited to the High Priest for punishment according to Jewish law[5] (qualifying this with care[6]); noting support for the record,[7] he still urges caution.[8]

The Maccabean parallel is dismissed as historically inadequate by Légasse[9] and Marshall,[10] but Bruce defends it with reference to a decree by Julius Caesar re-affirming all the previously held rights of the High Priest.[11] Kistemaker and Hendriksen likewise believe the High Priest actually had extradition authority.[12]

Dunn disputes the idea of formal jurisdiction,[13]  but notes the informal influence of the high priest and Sanhedrin over provincial synagogues was far higher.[14]

Bond[15] and Williams[16] note similarly that the letters would have carried influence despite their lack of formal weight.

Wallace and Williams approach the legal-historical background with care.[17] Observing the letters were addressed to the synagogues not local officials, they argue the matter was internal Jewish business in which Roman officials would not become involved.[18] Noting the apparent absence of Roman forces in Damascus at the time, they suggest this would have reduced the probability of Roman interference. [19]

Klauck and Bailey also view the letters as simply letters of introduction rather than legal documents with which to exercise authority over local officials,[20] and note no difficulty with the record. Oepke,[21]  Bond,[22] and Gaertner[23]  take a similar view.


[1] Acts 9: 1 Meanwhile Saul, still breathing out threats to murder the Lord’s disciples, went to the high priest 2 and requested letters from him to the synagogues in Damascus, so that if he found any who belonged to the Way, either men or women, he could bring them as prisoners to Jerusalem.

[2] ‘Neither the high priest nor the Jewish Sanhedrin in Jerusalem ever had such powers of jurisdiction. The persecution would have taken the regular process in the local synagogue:’, Köster, ‘Introduction to the New Testament’, volume 2, p. 107 (2006).

[3]neither the high priest nor the Sanhedrin had judicial authority outside the eleven toparchies of Judaea proper. Their moral authority might be persuasive, but they could not empower Paul to make arrests, particularly on the territory of a Roman province.’, Murphy-O’Connor, ‘Paul: a critical life’, p. 66 (1998).

[4] ‘The jurisdiction of the High Priest and the Sanhedrin would in fact have been limited to the eleven toparchies of Judaea.’, Légasse, ‘Paul’s pre-Christian Career according to Acts’, in Bauckham (ed.), ‘The Book of Acts in its Palestinian Setting’, p. 389 (1995).

[5] ‘The Roman legal system was not built on the territorial principle of law, but on the personal.64 This meant that a Roman citizen fell under Roman law wherever he was. In consequence, it may have been that the High Priest in Jerusalem could extend his jurisdiction to Jews in Damascus.65 That this practice was indeed prevalent is often argued on the basis of a passage from 1 Maccabees: 1 Macc 15, 16-21. Here, the Roman Consul Lucius writes to the Egyptian king Ptolemy (probably VIII) on the subject of the Jews: ‘if any scoundrels have fled to you from their country, hand them over to the High Priest Simon, so that he may punish them according to their law’ Peerbolte, ‘Paul the Missionary’, p. 154 (2003).

[6] ‘However, although the assumption is that this custom was still in use in Paul’s day, it is unclear whether this was correct’, ibid., p. 154.

[7] ‘Many students of the book of Acts nevertheless consider 9, 1-2 as evidence that Paul was sent as a shaliach by the Sanhedrin’, ibid., p. 154.

[8] ‘Still, a more cautious approach is to be preferred: we simply cannot decide with certainty on the historicity of Paul’s commissioning by the High Priest. It is a possibility, but remains far from certain.’, ibid., p. 154.

[9] ‘But, even supposing this letter is authentic,85 it is not addressed, like the letter of which Acts speaks, to the ‘synagogues’ but to a local ruler by the Roman authority. The case is therefore wholly different, as are the period (the events mentioned are supposed to have occurred in 139 BC) and the political situation: whereas at the time of Paul Judaea was a Roman province administered by a Roman governor, Simon, the brother of Judas Maccabeus, was a sovereign, autochthonous vassal of the Seleucids of Antioch.’, Légasse, ‘Paul’s pre-Christian Career according to Acts’, in Bauckham (ed.), ‘The Book of Acts in its Palestinian Setting’, p. 389 (1995).

[10] ‘Haenchen (p. 320 n.2) argues rightly that previous scholars have drawn unwarranted deductions from such passages as 1 Maccabees 15:15–21, which deals with a different and much earlier situation;’, Marshall, (1980). Vol. 5: ‘Acts: An introduction and commentary’ Tyndale New Testament Commentaries, volume 5, p. 178 (1980).

[11]Julius Caesar confirmed those rights and privileges anew to the Jewish nation (although Judaea was no longer a sovereign state), and more particularly to the high-priesthood.5 Luke’s narrative implies that the right of extradition continued to be enjoyed by the high priest under the provincial administration set up in A.D. 6. The followers of The Way whom Saul was authorized to bring back from Damascus were refugees from Jerusalem, not native Damascene disciples.’, Bruce, ‘The Book of the Acts’, New International Commentary on the New Testament, pp. 180-181 (1988); his source for the decree of Caesar is a passage by Josephus, ‘I also ordain, that he and his children retain whatsoever privileges belong to the office of high priest, or whatsoever favors have been hitherto granted them;’, Antiquities 14.195, in Whiston, ‘The Works of Josephus: Complete and unabridged’ (electronic ed. 1996).

[12] ‘The high priest served as head of the Sanhedrin, which as a legislative body had jurisdiction over the Jews living in Jerusalem, Palestine, and the dispersion. Thus the high priest had power to issue warrants to the synagogues in Damascus for the arrests of Christian Jews residing there (see 9:2; 22:5; 26:12).’, Kistemaker & Hendriksen, ‘Exposition of the Acts of the Apostles’, Baker New Testament Commentary, volume 17, p. 329 (1953-2001); as evidence they cite ‘Emil Schürer, The History of the Jewish People in the Age of Jesus Christ (175 B.C.–A.D. 135), rev. and ed. Geza Vermes and Fergus Millar, 3 vols. (Edinburgh: Clark, 1973–87), vol. 2, p. 218.’, p. 329.

[13] ‘the high priest had no formal jurisdiction over synagogues, least of all in other countries.’, Dunn, ‘Beginning from Jerusalem’, p. 337 (2009).

[14]But he had at least two considerable constraints which he could bring to bear on archisynagōgoi and synagogue elders. One was that he was responsible for much of the content and timing of lived-out Judaism; he and his councillors were the ultimate authority in matters of dispute, and it is not at all unlikely that Jerusalem authorities occasionally wrote to disapora synagogues to encourage them to maintain the traditions and possibly to take sides in some dispute on timing of festivals and the like.86 The high priest might even have been willing to claim jurisdiction over a ‘greater Judea’ which included Damascus. In any case, the high priest was not a person whose envoy could be lightly disregarded or dismissed with his mission unfulfilled. The other is that the Temple in Jerusalem held an amazing range of financial deposits for Jews at home and abroad; it was Judaism’s ‘central bank’. It is quite conceivable, therefore, that any requests were backed, explicitly or implicitly, with threat of financial sanctions.’, ibid., p. 337.

[15] ‘Writing letters to Diaspora communities was one of the high priest’s duties (see above, p. 47). Such letters would have had no formal weight (the high priest had no legal jurisdiction in Damascus, situated as it was in the Roman province of Syria), but his position as high priest would have conferred authority on his requests.’, Bond, ‘Caiphas: friend of Rome and judge of Jesus?’, p. 81 (2004); she also suggests ‘Second, and more probably, the rather vague reference to “the high priest” in 9:1-2 and in the flashbacks of 22:5 and 26:12 may be simply another example of Luke’s attempt to give opposition to Christians official backing.’, p. 81.

[16] ‘The letters to the synagogues (v. 2) would be a help, for though the Sanhedrin had no legal authority outside Judea, its reputation did give it some moral authority over the Jews of the Diaspora (see Sherwin-White, p. 100). Paul would also have had to seek the cooperation of the local magistrates, but the name of the Jewish Sanhedrin may have carried sufficient weight even with them for him to be confident of their acquiescence, if not their active assistance.’, Williams, ‘Acts’, New International Biblical Commentary, pp. 167-168 (1990).

[17]Unfortunately, we know very little about the internal affairs of Damascus  in Paul’s day. It is therefore difficult to know how to make sense of Paul’s commission from the High Priest to seize and carry to Jerusalem ‘any belonging to the Way’ (Acts 9, 2).’, Wallace & Williams, ‘The Three Worlds of Paul of Tarsus’, p. 163 (1998).

[18] ‘Since Acts says quite clearly that the letters Paul was carrying were to the synagogues at Damascus (9, 2) and not to the Gentile authorities, whatever he was doing must have been an entirely internal Jewish affair.‘, ibid., p. 163.

[19] ‘Since it is unlikely that the arrest and extradition to Judaea of dissenters was one of the privileges enjoyed by diaspora communities (for discussion see Wallace and Williams 1995:51-2), what Paul was engaged in must have been unauthorised; that is to say, kidnapping. So why was he not stopped? Such evidence as there is suggests that no Roman forces were stationed in Damascus (Millar 1993:37), so that unless an appeal was made to the governor, or serious disorder broke out, the Roman authorities would not have become involved. As for the city authorities, if the business was done discreetly without causing public disturbances they might well have taken the view that what went on in the Jewish community was none of their concern, especially if those involved were not citizens of Damascus, but incomers.‘, ibid., pp. 163-164.

[20] ‘As a persecutor of Christians Paul carried letters with him to gain admittance into the synagogues in Damascus as an otherwise unknown representative of the high priest and the Jewish elders (Acts 9:1-2; 22:5).’, Klauck & Bailey, ‘Ancient Letters and the New Testament: a guide to context and exegesis’, p. 76 (2006).

[21] ‘Oepke, ‘Probleme’, 403/426, who does not exclude a request from Paul to the High Priest, sees it, not as a mandate to arrest officially entrusted to Paul, but a letter like the sustati kai epoistolai to which 2 Cor. 3:1 refers.’, Légasse, ‘Paul’s pre-Christian Career according to Acts’, in Bauckham (ed.), ‘The Book of Acts in its Palestinian Setting’, p. 389 (1995).

[22] ‘The question is whether any evidence supports a situation in which the Sanhedrin had authority over synagogues so far from home.4 The point may be moot, however, in view of the fact that Luke does not say that the letters were papers of extradition. The letters may simply have been letters introducing Paul and his mission, as well as recommendations that such Jews be handed over to him. Such letters would carry no official authority to enforce the arrests.’, Gaertner, ‘Acts’, The College Press NIV Commentary (electronic ed. 1993).

[23] ‘It is possible that Caiphas supplied Saul with letters of recommendation to Diaspora synagogues (rather like those of 2 Cor 3:1), introducing him to their leaders, asking for help to root out troublemakers’, Bond, ‘Caiphas: friend of Rome and judge of Jesus?’, p. 81 (2004).

h1

The Merneptah Stele: Earliest evidence for Israel in Canaan?

April 21, 2011

Israel In Canaan

The Merneptah Stele is a pillar erected by Pharaoh Merneptah, recording his conquests in 13th century BCE Canaan.[1]  Among them, Merneptah records the Israelites, proving they were established in Canaan by then.[2]

Challenges

A minority of Biblical scholars have challenged the reading of the Merneptah Stele,[3] suggesting that it does not refer to the Israelites; representatives of this view include Gösta Werner Ahlström and Diana Edelman, [4] Thomas Thompson, [5] and Niels Peter Lemche.[6]

Scholarly Consensus

Rainey[7] has dismissed Ahlstrom and Edelman’s re-interpretation, and objected that they do not have the relevant training to read the inscription reliably.[8] Dever insists that the Stele ‘proves beyond any shadow of a doubt that there was a distinct ethnic group in Palestine before 1200, one that not only called itself “Israelite” but was known to the Egyptians as “Israelite.”’[9]

Whitelam acknowledges ‘It is well known that the Merneptah stela represents the earliest reference to Israel outside of the biblical texts’.[10] Oblath notes the stele ‘provides direct archaeological support for the early presence of Israelites in Canaan’.[11] Miller (II), states ‘The Israelite community may have been in Palestine before 1200 – the Merneptah Stele is evidence that it clearly was’.[12] [13]

Gottwald views the Merneptah Stele as part of the archaeological evidence demonstrating the authenticity of the Biblical description of the early Israelite population.[14] Long says ‘The text of the Merneptah stele portrays Israel as strong and associated with other powers and with major city-states of Canaan’. [15]

Miller and Hayes say the inscription ‘testifies to the existence of a population group, bearing the name “Israel”’. [16] Finkelstein and Silberman understand the Merneptah Stele as indicating ‘indicate that some group known as Israel was already in Canaan by that time’.[17]

Schley says ‘the Merneptah stele definitely identifies a non-settled group in Palestine as ‘Israel’ during the last decades of the thirteenth century’.[18] Long cites Edelman’s reference to acceptance of the Merneptah Stele as a reference to ‘some entity called Israel somewhere in Palestine in the late 13th century’, as part of a growing consensus on early Israelite history.[19]


[1] ‘The Merneptah Stele, also known as the Israel Stele, bears the oldest known written reference to Israel. Engraved with its current text in 1207 B.C.E., the 7.5-foot-high, black granite monolith was discovered in the ruins of Merneptah’s funerary temple in western Thebes in 1896. Most of its hieroglyphic text celebrates Merneptah’s defeat of the Libyans and their Sea Peoples allies in his fifth regnal year. The text’s last three lines, however, briefly mention a campaign into Canaan against the background of a pacified eastern Mediterranean political situation: “The rulers lie prostrate saying ‘Peace’; none raises his head among the Nine Bows [Egypt’s traditional enemies, by now a literary convention]. Plundering is for Tehenu [Libya]. Hatti is at peace. Canaan has been plundered into every sort of woe. Ashkelon has been overcome. Gezer has been captured. Yano’am was made non-existent. Israel is laid waste, (and) his seed is not. Hurru [Canaan] is become a widow for Egypt. All lands are united in peace.” The mention of Israel appears slightly to the left of center in the second line from the bottom. The glyphs include determinatives—signs indicating a word’s category—that classify Ashkelon, Gezer and Yano’am as city-states; but the determinative attached to Israel identifies it as a people, apparently not yet possessing a distinct city.’, Shanks, ‘Questions & Comments’, Biblical Archaeology Review (17.06), November/December 1991.

[2]Merneptah stele (ca 1225 B.C.) obviously establishes the people of Israel in Palestine and shows that they were known by that name in the 13th century. (Though some see here instead a reference to Jezreel, rather than Israel, the reading ya-si-r-˒i-ra seems clearly to indicate Israel; see W. F. Albright, Vocalization of the Egyptian Syllabic Orthography [1934], p. 34.)’, Lee,

Israel’, in Bromiley (ed.), ‘The International Standard Bible Encyclopedia, Revised’, pp. 907-908 (1992).

[3] ‘Revisionist scholars who do not accept the traditional reconstruction of the early history of Israel attempt to dismiss the reference to Israel in this text.’, Mazar, ‘The Israelite Settlement’, in Schmidt (ed.), ‘The Quest for the Historical Israel’, p. 93 (2007).

[4] ‘Sadly, one must make passing mention here of an attempt by G. W. Ahlstrom and Diana Edelman† to interpret “Israel” on the Merneptah Stele as a geographical entity (namely the central hill country of Canaan), despite the hieroglyphic determinative indicating that it denotes a people or tribe, an ethnic entity. In addition, Ahlstrom wants to abandon the correct reading, “Israel is desolated, his seed is not” for his own concoction: “Israel is laid waste, his grain is destroyed.”’, Rainey, ‘Scholars Disagree: Can You Name the Panel with the Israelites?’, Biblical Archaeology Review (17.06), November/December 1991.

[5] ‘ Hjelm and Thompson stress the poetic nature of the inscription, and suggest alternative identifications for ‘Israel’.’, Satterthwaite, P., & McConville, G. (2007). Exploring the Old Testament, Volume 2: The Histories (188–196).

[6] ‘He contends that Merneptah’s Israel may be simply a geographical designation or a political designation of an ethnic designation.’, Shanks, ‘Jerusalem’s Temple Mount: from Solomon to the golden Dome’, p. 154 (2007).

[7] Professor Emeritus of Ancient Near Eastern Cultures and Semitic Linguistics at Tel Aviv University and expert in Semitic.

[8]The phrase concerning the destruction of seed is a well-known Egyptian idiom in which “seed” means progeny, just as in the various Biblical passages about the “seed” of Abraham. Sometimes the determinative in Egyptian hieroglyphics for “seed” is the male genitals. Even though that determinative is missing from “seed” in the Merneptah Stele, the idiom always refers to progeny. Ahlstrom and Edelman have simply demonstrated that Biblical scholars untrained in Egyptian epigraphy should not make amateurish attempts at interpretation. A final qualification: By this demonstration, I do not mean to say that the “Israel” of the Merneptah Stele necessarily includes or is the equivalent of the 12-tribe nation depicted in the Bible. Some of the later tribes arrived in Canaan from different directions and perhaps at different times. However, the Merneptah Stele leaves no doubt that an ethnic group called “Israel” did exist in 1207 B.C.E.’, Rainey, ‘Scholars Disagree: Can You Name the Panel with the Israelites?’, Biblical Archaeology Review (17.06), November/December 1991.

[9] ‘First of all, we have not only the biblical tradition that calls them Israelites, but we also have the Merneptah Stele that proves beyond any shadow of a doubt that there was a distinct ethnic group in Palestine before 1200, one that not only called itself “Israelite” but was known to the Egyptians as “Israelite.” That need not be the same as later biblical Israel; but the label “Israelite,” which I want to apply to these early Iron I sites, is not one that I invented. It’s attested in the literary tradition, both biblical and non-biblical.’, Dever, ‘How to Tell a Canaanite from an Israelite’, in Shanks (ed.), ‘The Rise of Ancient Israel’, p. 54 (1992).

[10]Whitelam, ‘The Identity of Early Israel: The Realignment and Transformation of Late Palestine’, in Exum,. Vol. 40: ‘The Historical Books’, The Biblical Seminar, volume 40, p. (1997).

[11] ‘Erected in the late 13th century B.C.E., the stele provides direct archaeological support for the early presence of Israelites in Canaan. It is also the earliest extrabiblical text to mention Israel.’, Oblath, ‘The Exodus Itinerary Sites: Their Locations From the Perspective of the Biblical Sources’, Studies in Biblical Literature, volume 55,  p. 10 (2004).

[12] ‘The Israelite community may have been in Palestine before 1200 – the Merneptah Stele is evidence that it clearly was. Perhaps it was present quite some time before 1200, in fact.’, Miller (II), ‘Chieftains of the Highland Clans: A History of Israel in the Twelfth and Eleventh Centuries B.C.’, The Bible in Its World, p. xiv (2005).

[13] ‘The Merneptah Stele is direct positive evidence that the term “Israel” was used for some entity in the highlands of Palestine in the parlance of Late Bronze IIb sources’, ibid., p. 2.

[14] ‘What does appear to be established by the Merneptah stela and the archaeology of the highlands is that a population of cultivators and herders, at least some of whom bore the name Israel, lived in the regions of Canaan where the state of Israel subsequently arose, and furthermore that the biblical characterization of this population as politically decentralized and socially linked in village and kin arrangements is authentic’, Gottwald, ‘The Politics of Ancient Israel’, p. 164 (2001).

[15] Long, ‘Israel’s Past in Present Research: Essays on Ancient Israelite Historiography’, Sources for Biblical and Theological Study Old Testament Series p. 505 (1999).

[16]Thus the inscription testifies to the existence of a population group, bearing the name “Israel” and possibly tribal in structure, living in Canaan about 1230 B.C.E.’, Miller & Hayes, ‘A History of Ancient Israel and Judah’, p. 68 (1986).

[17] ‘Second, and perhaps most important, the earliest mention of Israel in an extrabiblical text was found in Egypt in the stele describing the campaign of Pharaoh Merneptah – the son of Ramesses II – in Canaan at the very end of the thirteenth century BCE. The inscription tells of a destructive Egyptian campaign into Canaan, in the course of which a people named Israel were decimated to the extent that the pharaoh boasted that Israel’s “seed is not!” The boast was clearly an empty one, but it did indicate that some group known as Israel was already in Canaan by that time. In fact, dozens of settlements that were linked with the early Israelites appeared in the hill country around that time.’, Finkelstein & Silberman, ‘The Bible Unearthed: Archaeology’s New Vision of Ancient Israel and the Origin of Sacred Texts’, p. 57 (2001).

[18] ‘And contrary to Finkelstein’s assertion that ‘there is no unequivocal archaeological evidence that the Israelite settlement began as early as the 13th century B.C.’, the Merneptah stele definitely identifies a non-settled group in Palestine as ‘Israel’ during the last decades of the thirteenth century.’, Schley, ‘Shiloh: A Biblical City in Tradition and History’, p. 79 (1989).

[19] ‘In a recent volume of the Scandanavian Journal of the Old Testament dedicated to the question of the emergence of Israel in Canaan, the volume’s editor, Diana Edelman, points to four areas of growing consensus: (1) that beginning in the Late Bronze Age and continuing through the Iron I period “population shifts and displacements” were taking place in Canaan, the net result of which was “the growth of new settlements in the Cisjordanian highlands”; (2) that “the Merneptah Stele indicates the existence of some entity called Israel somewhere in Palestine in the late 13th century”; (3) that “Israel is somehow to be related to the surge in small settlements in the highlands during the end of the Late Bronze – Iron I periods,” though “how this relationship is to be understood remains problematic”; (4) that “the biblical texts must be used with great caution in reconstructing the history of Israel’s origins and prestate conditions.”’, Long, ‘The Art of Biblical History’, p. 164 (1994).

h1

How tall was Goliath?

January 30, 2011

According to the Masoretic Text of the Old Testament[1] Goliath was six cubits and a span tall, 3.2 metres (9 feet, 6 inches), if measured by the 18 inch cubit, and 3.5 metres (just over 11 feet), if the 21 inch cubit is used. This is a height which is not only highly unlikely for any Iron Age man,[2] but far beyond what would have been considered a giant at the time.

The New English Translation (2005), describes Goliath’s height as just under seven feet, significantly below the traditional height.[3] A footnote in the NET explains the textual basis on which the translation committee made its decision.[4]

The evidence of the Masoretic Text is dated very late,[5] though this reading can be found in some earlier Greek translations of the Old Testament,[6] as well as the Latin Vulgate.[7] However, the textual evidence for a shorter height is more significant, being found in the oldest Greek and Hebrew texts.[8] [9]

  • The LXX:[10] 6 feet, 9 inches
  • 4QSama: [11] 6 feet, 9 inches
  • Josephus:[12] 6 feet, 9 inches
  • Lucian recension:[13] 6 feet, 9 inches
  • Codex Vaticanus:[14] 6 feet, 9 inches
  • Codex Alexandrinus:[15] 6 feet, 9 inches

Some object that Goliath’s weapons and armour make little sense for a warrior shorter than the traditional reading.[16] However, such objections are not well founded.[17] [18]

Modern scholarship prefers the reading of the earliest texts.[19] A man of this height would still have been considered a giant in the Iron Age.[20] Some reference sources cite medical conditions such as acromegaly[21] which could have resulted in the height in the Masoretic Text,[22] a suggestion found even in some professional medical works.[23]

Although possible, it is unlikely such a man would have been an effective warrior given the disabilities typical of such conditions,[24] and since acromegaly affects humans in middle age it would have been very rarely seen given the average Iron Age life expectancy of less than 40 years.[25] [26]


[1] The medieval Hebrew text on which almost all modern English translations of the Old Testament are based.

[2] ‘Archaeology has shown that the heroes buried in the “royal tombs” at Mycenae were 1.76-1.80 mtr. tall, while the height of the average man at that period (according to the skeletons excavated) was 1.64 mtr. both in the Aegean lands and in Canaan.’, Margalith, ‘The Sea Peoples in the Bible’, p. 49 (1994).

[3] 1 Samuel 17:4 Then a champion came out from the camp of the Philistines. His name was Goliath; he was from Gath. He was close to seven feet tall.

[4]7 tc Heb “his height was six cubits and a span” (cf. KJV, NASB, NRSV). A cubit was approximately eighteen inches, a span nine inches. So, according to the Hebrew tradition, Goliath was about nine feet, nine inches tall (cf. NIV, CEV, NLT “over nine feet”; NCV “nine feet, four inches”; TEV “nearly 3 metres”). However, some Greek witnesses, Josephus, and a manuscript of 1 Samuel from Qumran read “four cubits and a span” here, that is, about six feet, nine inches (cf. NAB “six and a half feet”). This seems more reasonable; it is likely that Goliath’s height was exaggerated as the story was retold. See P. K. McCarter, I Samuel (AB), 286, 291.’, NET, footnote on 1 Samuel 17:4 (1st ed. 2005).

[5]‘What is the evidence for the variant which reads six cubits and a span (9’9″)? All of the manuscripts of the MT have this reading. However, one should keep in mind that the earliest MT manuscript evidence that we have for 1 Samuel is the Aleppo Codex, produced in AD 935. Likewise, the Leningrad Codex, the Hebrew manuscript on which BHK and BHS are based, and thus the major Hebrew text on which most of our English OT versions are based, was completed in AD 1010. Thus there is no extant Hebrew text any earlier than AD 935 that puts Goliath at six cubits and a span.’, Hays, ‘Reconsidering the Height of Goliath’, Journal of the Evangelical Theological Society (48.5.705), (2005).

[6]However, the variant reading “six cubits and a span” is probably much older than these MT manuscripts. Symmachus, for example, reflects the six cubits and a span height in his translation. Symmachus was a Jew who around AD 200 prepared a Greek translation of the OT for the Jewish community in Caesarea of Palestine. His goal was to produce a Greek translation that was an accurate translation of the Hebrew.7 The fact that he gives Goliath’s height as six cubits and a span is strong evidence that this variant reading was already present in the proto-MT or Vorlage to the MT, that is, the Hebrew text tradition that by this time had become the standardized text of the Jews. Likewise, Origen, in his Greek translation found in the fifth column of his Hexapla, includes the longer version of 1-2 Samuel found in the MT and also lists Goliath’s height as six cubits and a span.’, ibid., p. 705.

[7]Jerome’s fourth-century AD Latin translation (the Vulgate), which appears to follow the Hebrew proto-MT faithfully, likewise puts Goliath at six cubits and a span. The Vulgate gradually grew in popularity in the Western churches and eventually became the “received text” for the Western churches, thus codifying the 9’9″ giant into Western culture.’, ibid., p. 705.

[8]‘However, the textual evidence for the variant reading of “four cubits and a span” is significantly stronger.’, ibid., p. 706; Hays (pp. 705-706), notes that Codex Venetus, an 8th century manuscript (also known as ‘Codex Venetus 5’, a Greek manuscript of the Old and New Testaments), has the reading ‘five cubits and a span’ (8 feet 3 inches), but dismisses this as a rogue reading unsupported by any other textual evidence.

[9]‘Thus the textual witnesses for the variant that cites Goliath’s height at four cubits and a span (6’9″) include: (1) 4QSama, the oldest extant Hebrew manuscript for this text; (2) Vaticanus, the oldest complete Greek codex of the Bible; (3) Josephus, a non-biblical first-century ad reference; (4) the Lucian Greek recension, a third-century ad witness; and (5) other early codices such as Alexandrinus.’, ibid., p. 705.

[10]Commonly known as the ‘Septuagint’, a Greek translation of the Old Testament, completed no later than the mid-2nd century BCE.

[11]One of the Hebrew copies of the book of Samuel found among the ‘Dead Sea Scrolls’, this text contains 1-2 Samuel, and although parts of the text are damaged, 1 Samuel 17:3-6 is not; this text dates to around 50 BCE and is the oldest Hebrew copy of this part of 1 Samuel, so it is significant that it has the shorter height.

[12] Josephus, ‘Antiquities of the Jews’, 6.171 (1st century CE); Josephus was a 1st century Jewish historian, and although it is not likely that Josephus had access to historical records of Goliath other than the Biblical texts, the height he gives demonstrates that this was already recorded in the Hebrew Bible by his day, long before the taller height.

[13] This is an early 3rd century revision of the LXX text.

[14] A significant 4th century Greek manuscript containing almost all of the Old and New Testaments.

[15] A significant 5th century Greek manuscript containing almost all of the Old and New Testaments.

[16] ‘Some commentators point to the huge size of Goliath’s spear, “with a shaft like a weaver’s beam” and “a point that weighed six hundred shekels” (about 15 pounds) and conclude that this huge spear size argues for a nine and a half foot tall Goliath. Likewise, they underscore the massive weight of his armor (125-130 pounds) and conclude that the size of the armor fits better with a 9’9″ giant than with a 6’9″ warrior.’, Hays, ‘Reconsidering the Height of Goliath’, Journal of the Evangelical Theological Society (48.5.708), (2005).

[17] ‘However, first of all, the text does not say that Goliath’s spear was as big or as long or as heavy as a weaver’s beam. It just says that the “shaft” (lit. “arrow”) of his spear was as (D) a weaver’s beam. Yagael Yadin points out that the comparison between Goliath’s spear and a weaver’s beam is not saying that the shaft of the spear was as big as a large timbered crossbeam of a loom. Rather, the narrator is describing a looped cord or rope that was attached to the spear that enabled a warrior to throw it harder and further. This looped cord looked somewhat like the cord loops of a weaver’s beam; thus the analogy.15 It has nothing to do with the size of his spear.16 Thus it also has no bearing on the size of Goliath. Likewise, the six hundred shekel (fifteen pounds) weight of the iron spearhead certainly would not require a ten foot tall giant to throw it..’, ibid., p. 708; spears were also commonly used to thrust at short range (requiring less effort), rather than thrown (note Goliath does not throw his spear).

[18]Yadin provides pictures of a real weaver’s beam as

well as ancient paintings of soldiers from the ancient Near East holding such looped weapons.8 His evidence is so convincing that, as I point out, the majority of commentators on 1 Samuel follow Yadin.9’, Hays, ‘A Response to Clyde Billington’, Journal of the Evangelical Theological Society (50.3.515), (2007).

[19] ‘In the MT, Goliath’s height is given as “six cubits and a span” (1 Sam 17:4), which would be about nine feet and nine inches, a true giant. However, LXX Codex Vaticanus and the Lucianic recension, as well as 4QSama and Josephus (Ant 6.171) all give Goliath’s height as “four cubits and a span,” about six feet and nine inches. Since the expected tendency would be to exaggerate the height of David’s opponent, the latter reading, according to which Goliath would still be a giant (albeit among men), is to be preferred (see McCarter 1 Samuel AB, 286).’, Ehrlich, ‘Goliath’, in Freedman (ed.), ‘The Anchor Yale Bible Dictionary’, volume 2, p. 1073 (1996).

[20] A 6 foot 9 inch Goliath (2.05 metres), would have been at least 20 centimetres taller than the heroes found in Iron Age Mycenaen tombs, and 30-40 centimetres taller than the average Iron Age man; this proves that even the heroes of the time were only 10-15 centimetres taller than the average, and a man 30 centimetres taller than these heroes would have been a real giant, almost 40 centimetres taller than the average man (Saul need not have been more than 6 feet tall to be head and shoulders above all in Israel, and at 6 feet 9 inches Goliath would still have been significantly taller).

[21] A pituitary gland disorder resulting in giantism.

[22] ‘However, D. Kellermann suggests that Goliath’s symptoms in 1 Samuel 17 match those of pathological gigantism (a pituitary condition known as acromegaly), including a tunnel-vision type of visual defect, which David presumably took advantage of in defeating him. If so, then the MT reading need not be considered an exaggeration.’, Li, ‘Goliath’, in Arnold & Williamson (eds.), ‘Dictionary of the Old Testament: Historical books’, p. 356 (2005).

[23] ‘The biblical giant Goliath may have been afflicted with acromegaly.’, Ember & Ember, ‘Encyclopedia of Medical Anthropology’, p. 392 (2004).

[24] Aside from various deformities, digestive problems, and high blood pressure, premature death is also a danger with acromegaly.

[25] Nevertheless, evidence from an Egyptian tomb dated well over 1,000 years before Goliath does indicate that acromegaly could occur earlier in life even during the Bronze Age; ‘The remains of a large adult male, probably in his late 20s or early 30s, from a Fifth Dynasty tomb (2494–2345 BC)…This individual exhibits characteristics of pituitary gigantism’, Mulhern, ‘A probable case of gigantism in a fifth Dynasty skeleton from the Western Cemetery at Giza, Egypt’, International Journal of Osteoarchaeology (15.4.261), (2005).

[26] The Bible mentions a number of champions from Gath, some with physical abnormalities (2 Samuel 21:20, 1 Chronicles 20:6), all apparently related by descent from ‘Rapha’, possibly ‘the giant’ (2 Samuel 21:16, 18, 20, 22, 1 Chronicles 20:4, 6, 8), so it is more likely that Goliath’s height was the result of a family genetic trait than the result of a later medical condition.

h1

Article: The Historicity of the book of Acts (3)

December 28, 2010

Peter’s address: Acts 4:4

Robert Grant claimed that the population of Jerusalem was too small for 5,000 converts to Christianity.[1] Grant’s estimate of the population of Jerusalem relied on an influential study by Jeremias in 1943, [2] [3] but did not mention that Jeremias calculated a far higher population figure for festival seasons such as passover, at which he estimated Jerusalem would contain up to 125,000 pilgrims.[4]

Furthermore the lower estimate of Jeremias is significantly lower than the lowest of the moderate to high estimates made by Wilkinson in 1974 (70,398 under Herod the Great),[5] Broshi in 1976 (60,000),[6] Maier in 1976 (50,000, with three times that many during festival seasons),[7] and Levine in 2002 (60,000-70,000).[8]

Accordingly, Cousland notes that ‘recent estimates of the population of Jerusalem suggest something in the neighbourhood of a hundred thousand’.[9] Estimates for the number of Christians in the Roman empire by the end of the first century range widely from 7,500 to more than 50,000. [10] [11] [12]

Article here.


[1] Grant ‘A Historical Introduction to the New Testament, p. 145 (1963).

[2] Jeremias, ‘Die Einwohnerzhal Jerusalems z. Zt. Jesu’, ZDPV, 63, pp. 24-31 (1943).

[3] ‘Jeremias, for instance has estimated that there was a population of 25,000 in first century Jerusalem,’, Rocca, ‘Herod’s Judaea: A Mediterranean State in the Classical World’, p. 333 (2008).

[4] ‘Thus one would arrive at 125,000 festival pilgrims.’, Reinhardt, ‘The Population Size of Jerusalem and the Numerical Growth of the Jerusalem Church’, in Bauckham (ed.), ‘The Book of Acts in its Palestine Setting’, p. 261 (1995).

[5] Wilkinson, ‘Ancient Jerusalem, Its Water Supply and Population’, PEFQS 106, pp. 33-51 (1974).

[6] ‘This also gives a figure of around 60,000 at the time of the first Christians.’, Reinhardt, ‘The Population Size of Jerusalem and the Numerical Growth of the Jerusalem Church’, in Bauckham (ed.), ‘The Book of Acts in its Palestine Setting’, p. 247 (1995).

[7] Maier, ‘First Christians: Pentecost and the Spread of Christianity’, p. 22 (1976).

[8]‘According to Levine, because the new area encompassed by the Third Wall was not densely populated, assuming that it contained half the population of the rest of the city, there were between 60,000 and 70,000 people living in Jerusalem.’, Rocca, ‘Herod’s Judaea: A Mediterranean State in the Classical World’, p. 333 (2008).

[9] Cousland, ‘The Crowds in the Gospel of Matthew’, p. 60 (2002).

[10] Stark, ‘The Rise of Christianity’, pp. 6-7 (1996); this influential study dominates the literature, but has been questioned as it involves projecting backwards from fourth century estimates.

[11] Wilken, ‘The Christians as the Romans Saw Them’, p. 31 (1984).

[12] ‘Estimates for the number of Christians by 100 C.E. range from as low as 7,500 to upwards of 50,000 out of the approximately sixty million inhabitants of the Roman Empire.’, Novak, ‘Christianity and the Roman Empire: background texts’, pp. 12-13 (2001).